You are on page 1of 76

Total Marks : 200

Test-2 (Subject)
( INSTA Prelims Test Series 2021 )

1. Consider the following statements regarding Vedanta School of Thought


1. It believes in the argument of brahma and indestructible atma.
2. It denies the theory of Punarjanama or rebirth
3. Shankaracharya considers brahma to possess certain attributes

Which of the statements given above is/are correct?


A. 1 only
B. 1 and 3 only
C. 2 only
D. 2 and 3 only

Correct Answer : A

Answer Justification :

Vedanta is made of two words- ‘Veda’ and ‘ant’, i.e. the end of the Vedas. This school upholds
the philosophies of life as elaborated in the Upanishads

This argument considers atma and brahma to be the same and if a person attains the
knowledge of the self, he would automatically understand brahma and would achieve
salvation. This argument would make brahma and atma indestructible and eternal.
Hence Statement 1 is correct.

The Vedanta theory also gives credence to the Theory of Karma. The theory believes
in Punarjanama or rebirth.They also argued that a person would have to bear the brunt of
their actions from the previous birth in the next one. Hence Statement 2 is incorrect.

Ramanujacharya considers brahma to possess certain attributes. He advocates loving


the faith and practicing devotion as the path to attain salvation. Hence Statement 3 is
incorrect.

2. Which of the following are considered as Triratnas under Jainism?


1. Right faith
2. Right livelihood
3. Right knowledge
4. Right speech
5. Right conduct

Select the correct answer using the code given below:


A. 1, 2 and 3 only
B. 2, 3 and 4 only
C. 1, 3 and 5 only
D. 3, 4 and 5 only

www.insightsactivelearn.com 1
Total Marks : 200
Test-2 (Subject)
( INSTA Prelims Test Series 2021 )

Correct Answer : C

Answer Justification :

Teachings of Mahavira

The three principles of Jainism, also known as Triratnas (three

gems), are:

1. right faith
2. right knowledge
3. right conduct.

Hence, statement 2 and 4 are incorrect.

Right faith is the belief in the teachings and wisdom of Mahavira. Right Knowledge is the
acceptance of the theory that there is no God and that the world has been existing without a
creator and that all objects possess a soul. Right conduct refers to the observance of the five
great vows:

1. not to injure life


2. not to lie
3. not to steal
4. not to acquire property
5. not to lead an immoral life.

Both the clergy and laymen had to strictly follow the doctrine of ahimsa. Mahavira regarded all
objects, both animate and inanimate, have souls and various degrees of consciousness. They
possess life and feel pain when they are injured.

Mahavira rejected the authority of the Vedas and objected to the Vedic rituals. He advocated a
very holy and ethical code of life. Even the practice of agriculture was considered sinful as it
causes injury to the earth, worms and animals.

3. Consider the following statements regarding the dancing figure of Shiva as Nataraja
1. The right leg is suppressing the Apasmara, the demon of ignorance.
2. The left leg is in bhujangatrasita stance, which represents tirobhava, that is kicking away the
veil of maya or illusion.
3. The main right hand is held in dola hasta and connects with the abhaya hasta of the left hand.

www.insightsactivelearn.com 2
Total Marks : 200
Test-2 (Subject)
( INSTA Prelims Test Series 2021 )

Which of the statements given above is/are correct?


A. 1 and 2 only
B. 1 and 3 only
C. 2 and 3 only
D. 1, 2 and 3

Correct Answer : A

Answer Justification :

Nataraja

Shiva is associated with the end of the cosmic world with which this dancing Position of
Nataraja is associated.

In this Chola period bronze sculpture he has been shown balancing himself on his
right leg and suppressing the apasmara, the demon of ignorance or forgetfulness,
with the foot of the same leg. At the same time he raises his

left leg in bhujangatrasita stance, which represents tirobhava, that is kicking away
the veil of maya or illusion from the devotee’s mind. His four arms are outstretched and
the main right hand is posed in abhaya hasta. The upper right holds the damaru his favourite
musical instrument to keep on the beat tala. The upper left hand carries a flame while the
main left hand is held in dola hasta and connects with the abhaya hasta of the right
hand. Hence, statement 3 is incorrect.

His hair locks fly on both the sides touching the circular jvala mala or the garland of flames
which surrounds the entire dancing figuration.

4. Istanbul Convention, sometimes seen in the news, is related to

A. Child labour
B. Terrorism
C. Disaster Management
D. Violence against women and domestic violence

Correct Answer : D

Answer Justification :

Istanbul Convention:

www.insightsactivelearn.com 3
Total Marks : 200
Test-2 (Subject)
( INSTA Prelims Test Series 2021 )

• It is also called as the Council of Europe Convention on preventing and combating


violence against women and domestic violence.

• The treaty is the world’s first binding instrument to prevent and tackle violence against
women.

• It is the most comprehensive legal framework that exists to tackle violence against
women and girls, covering domestic violence, rape, sexual assault, female genital mutilation
(FGM), so-called honour-based violence, and forced marriage.

• The Convention sets minimum standards for governments to meet when tackling
violence against women.

• When a government ratifies the Convention, they are legally bound to follow it.

5. Consider the following statements regarding Yoga School of Philosophy


1. It believes that human being can achieve salvation by combining meditation and physical
application of yogic techniques.
2. It is argued that these techniques lead to the release of the Purusha from the Prakriti and
would eventually lead to salvation.

Which of the statements given above is/are correct?


A. 1 only
B. 2 only
C. Both 1 and 2
D. Neither 1 nor 2

Correct Answer : C

Answer Justification :

Both the statements are correct.

The Yoga school literally means the union of two major entities. They argue that human
being can achieve salvation by combining meditation and physical application of
yogic techniques.

It is argued that these techniques lead to the release of the Purusha from the Prakriti
and would eventually lead to salvation. This origin of Yoga and the school have been
expounded in the Yogasutra of Patanjali that is attributed tentatively to the 2nd century BCE.

The physical aspect of this school deals primarily with exercises in various postures that are
also called asanas. There are several types of breathing exercises that are called pranayams.

www.insightsactivelearn.com 4
Total Marks : 200
Test-2 (Subject)
( INSTA Prelims Test Series 2021 )

6. Consider the following statements regarding Teachings of Buddha


1. Buddha rejects the existence of god.
2. Buddha taught that the soul does not exist.
3. Buddha laid great emphasis on the law of karma.

Which of the statements given above is/are correct?


A. 1 and 2 only
B. 1 and 3 only
C. 2 and 3 only
D. 1, 2 and 3

Correct Answer : C

Answer Justification :

Teachings of Buddha

The Four Noble Truths of Buddha are:

- The world is full of suffering.

- The cause of suffering is desire.

- If desires are gotten rid off, suffering can be removed.

- This can be done by following the Eightfold Path.

The Eightfold Path consists of right view, right resolve, right speech, right conduct, right
livelihood, right effort, right mindfulness and right concentration. Buddha neither accepts god
nor rejects the existence of god. Hence, statement 1 is incorrect.

He laid great emphasis on the law of karma. He argued that the condition of man in this life
depends upon his own deeds. He taught that the soul does not exist. However, he emphasized
Ahimsa. By his love for human beings and all living creatures, he endeared himself to all.

7. Consider the following statements regarding the statue of Gomateshwara


1. It is the granite statue of Lord Mahaveer
2. It is the world’s tallest monolithic free-standing structure.
3. It was commissioned by Camundaraya, Prime Minister of the Ganga Kings of Mysore.

Which of the statements given above is/are correct?


A. 1 and 2 only
B. 1 and 3 only
C. 2 and 3 only
D. 1, 2 and 3

www.insightsactivelearn.com 5
Total Marks : 200
Test-2 (Subject)
( INSTA Prelims Test Series 2021 )

Correct Answer : C

Answer Justification :

Karnataka has a rich heritage of Jain shrines and at Sravana Belagola the famous statue of
Gomateshwara, the granite statue of Lord Bahubali which stands eighteen metres or fifty-
seven feet high, is the world’s tallest monolithic free-standing structure. Hence,
statement 1 is incorrect.

It was commissioned by Camundaraya, the General-in-Chief and Prime Minister of


the Ganga Kings of Mysore.

8. Consider the following statements regarding National Commission for Protection of Child Rights (NCPCR)
1. It is a non-statutory body set up by the executive resolution.
2. Chairperson and members of the commission are appointed by Central Government for five
years.
3. It works under the administrative control of the Ministry of Women & Child Development.

Which of the statements given above is/are correct?


A. 3 only
B. 2 and 3 only
C. 1 and 2 only
D. 1, 2 and 3

Correct Answer : A

Answer Justification :

National Commission for Protection of Child Rights (NCPCR):

• The National Commission for Protection of Child Rights (NCPCR) is a statutory body
established under the Commission for Protection of Child Rights Act in December
2005. Hence, statement 1 is incorrect.

• The commission works under the aegis of Ministry of Women and Child
development. Hence, statement 3 is correct.

• Its Mandate is “to ensure that all Laws, Policies, Programmes, and Administrative
Mechanisms are in consonance with the Child Rights perspective as enshrined in the
Constitution of India and the UN Convention on the Rights of the Child.

• Definition of a child: As defined by the commission, child includes those up to the age of
18 years.

www.insightsactivelearn.com 6
Total Marks : 200
Test-2 (Subject)
( INSTA Prelims Test Series 2021 )

Composition:

•This commission has a chairperson and six members of which at least two should be women.

• All of them are appointed by Central Government for three years. Hence, statement
2 is incorrect.

• The maximum age to serve in the commission is 65 years for Chairman and 60 years for
members.

9. Consider the following statements regarding Charvaka School of Philosophy/ Lokayata School of Philosophy
1. Man is the centre of all activities of this philosophy and he should enjoy himself as long as he
lives.
2. Kapila laid the foundation stone of this school
3. It argued for a complete disregard of any world beyond this world that was inhabited by a
person.

Which of the statements given above is/are correct?


A. 2 only
B. 1 and 3 only
C. 2 and 3 only
D. 1 only

Correct Answer : B

Answer Justification :

Brihaspati laid the foundation stone for Charvaka School of Philosophy and it was
supposed to be one of the earliest schools that developed a philosophical theory. Hence
Statement 2 is incorrect.

The word ‘Lokayata’ also meant a keen attachment to the physical and material world (loka).
They argued for a complete disregard of any world beyond this world that was
inhabited by a person. Hence Statement 3 is correct.

Some of their main teachings are:

They argued against Gods and their representatives on the earth – the priestly class. They
argued that a Brahman manufactures false rituals so as to acquire gifts (dakshina) from the
followers.

Man is the centre of all activities and he should enjoy himself as long as he lives. He
should consume all earthly goods and indulge in sensual pleasure. Hence Statement 1 is
correct.

www.insightsactivelearn.com 7
Total Marks : 200
Test-2 (Subject)
( INSTA Prelims Test Series 2021 )

10. Which of the following is/are the causes for the Decline of Buddhism in India?
1. The revival of Brahmanism and the rise of Bhagavatism.
2. The Buddhists began to adopt Sanskrit, the language of the elite.
3. The practice of idol worship.

Which of the statements given above is/are correct?


A. 1 and 2 only
B. 1 and 3 only
C. 2 and 3 only
D. 1, 2 and 3

Correct Answer : D

Answer Justification :

Causes for the Decline of Buddhism in India

The revival of Brahmanism and the rise of Bhagavatism led to the fall of popularity of
Buddhism. The use of Pali, the language of the masses as the language of Buddhism was given
up from the 1st century A.D. The Buddhists began to adopt Sanskrit, the language of the
elite. After the birth of Mahayana Buddhism, the practice of idol worship and making
offerings led to the deterioration of moral standards. Moreover, the attack of the Huns in 5th
and 6th centuries and the Turkish invaders in 12th century destroyed the monasteries. All
these factors contributed to the decline of Buddhism in India.

Hence, option (d) is correct.

11. Consider the following statements


1. Chaitya were residential halls for the Buddhist and Jain monks.
2. Vihars were mainly quadrangular chambers with flat roofs and used as prayer halls.

Which of the statements given above is/are correct?


A. 1 only
B. 2 only
C. Both 1 and 2
D. Neither 1 nor 2

Correct Answer : D

Answer Justification :

The construction of rock caves continued as in the Mauryan period. However, this period saw

www.insightsactivelearn.com 8
Total Marks : 200
Test-2 (Subject)
( INSTA Prelims Test Series 2021 )

the development of two types of rock caves – Chaitya and Vihar. While the Vihars were
residential halls for the Buddhist and Jain monks and were developed during the time of the
Mauryan Empire, the Chaitya halls were developed during this time. Hence, statement 1 is
incorrect.

Chaityas were mainly quadrangular chambers with flat roofs and used as prayer halls.
Hence, statement 2 is incorrect.

The caves also had open courtyards and stone screen walls to shield from rain. They were also
decorated with human and animal figures.

12. Consider the following statements regarding Nagarjunasagar Srisailam Tiger Reserve
1. The reserve is spread over in the regions of Andhra Pradesh and Telangana only.
2. Krishna River cuts its basin through the reserve.

Which of the statements given above is/are correct?


A. 1 only
B. 2 only
C. Both 1 and 2
D. Neither 1 nor 2

Correct Answer : C

Answer Justification :

Both the statements given above are correct.

Nagarjunasagar Srisailam Tiger Reserve (NSTR):

It is the largest Tiger Reserve in India. The reserve spreads over five districts in Andhra
Pradesh and Telangana. The area consists mostly of the Nallamala Hills. The Krishna
River cuts its basin almost 200 m (660 ft) deep over a distance of 130 km (81 mi)
through the reserve.

13. Consider the following statements regarding Indian Cinema


1. The first talking film was Alam Ara, which was produced by the Dadasaheb Phalke.
2. The enforcement of the censorship lies with Central Government.
3. All the films that are dubbed from one language to another have to get a fresh film certificate.

Which of the statements given above is/are correct?


A. 1 only
B. 2 and 3 only
C. 3 only

www.insightsactivelearn.com 9
Total Marks : 200
Test-2 (Subject)
( INSTA Prelims Test Series 2021 )

D. 1 and 3 only

Correct Answer : C

Answer Justification :

The first talking film was Alam Ara, which was produced by the Imperial Film
Company and directed by Ardeshir Irani. It was screened at Majestic Cinema in Bombay in
1931. Hence Statement 1 is incorrect.

All films have to get a Censor Board certificate. Even the foreign films that are imported to
India have to get a CBFC certification. All the films that are dubbed from one language to
another have to get a fresh certificate to ensure that the language change is not
offensive in any manner. Hence Statement 3 is correct.

Although the certification of films is a subject under the Central Government, the
enforcement of the censorship in their respective domains lies with the State
governments. Hence Statement 2 is incorrect.

14. Arrange the following temples chronologically from earliest to the latest
1. Lakshmana temple of Khajuraho
2. Durga temple at Aihole
3. Ananta Vasudeva temple, Bhubaneswar
4. Virupaksha temple

Select the correct answer using the code given below:


A. 1 2 4 3
B. 2 1 4 3
C. 2 4 1 3
D. 3 1 2 4

Correct Answer : C

Answer Justification :

The Durga temple at Aihole is an apsidal temple of about 550 A.D.

The Lakshmana temple of Khajuraho, dedicated to Vishnu, was built in 954 by the Chandela
king, Dhanga.

Among the later shrines of Bhubaneswar the Ananta Vasudeva temple, founded in 1278, is
remarkable in more ways than one. It is the only temple dedicated to Vaishnava worship at this

www.insightsactivelearn.com 10
Total Marks : 200
Test-2 (Subject)
( INSTA Prelims Test Series 2021 )

predominantly Shiva site and stands on an ornate platform terrace.

The great Virupaksha temple, dedicated to Siva as Lokesavara, by the queen of Vikramaditya II
is datable to 740 A.D., which was most likely built by workmen brought from Kanchipuram,
and in direct imitation of the Kailasanath at Kanchipuram.

Hence, option (C) is correct.

15. Consider the following statements regarding Mathura School of art


1. The Mathura School showed a striking use of symbolism in the images.
2. The halo around the head of Buddha is smaller than in Gandhara School and was decorated
with geometrical patterns.
3. The sculptures of Mathura School were made using spotted red sandstone.

Which of the statements given above is/are correct?


A. 1 and 2 only
B. 1 and 3 only
C. 2 and 3 only
D. 1, 2 and 3

Correct Answer : B

Answer Justification :

Mathura School

The Mathura School flourished on the banks of the river Yamuna in the period between 1st
and 3rd centuries B.C. The sculptures of the Mathura School were influenced by the stories and
imageries of all three religions of the time – Buddhism, Hinduism and Jainism. The images
were modelled on the earlier Yaksha images found during the Mauryan period. The Mathura
School showed a striking use of symbolism in the images. The Hindu Gods were represented
using their avayudhas. For example, Shiva is shown through linga and mukhalinga. Similarly,
the halo around the head of Buddha is larger than in Gandhara School and decorated
with geometrical patterns. Hence, statement 2 is incorrect.

Buddha is shown to be surrounded by two Bodhisattavas – Padmapani holding a lotus and


Vajrapani holding a thunderbolt.

The sculptures of Mathura School were made using spotted red sandstone.

Buddha is shown in delighted mood with a smiling face. The body symbolizes masculinity,
wearing tight dress. The face and head are shaven. Buddha is seated in padmasana with

www.insightsactivelearn.com 11
Total Marks : 200
Test-2 (Subject)
( INSTA Prelims Test Series 2021 )

different mudras and his face reflects grace. A similar protuberance is shown on the head.

16. Consider the following statements regarding Foreign Contribution (Regulation) Act (FCRA)
1. It is implemented by the Ministry of Finance
2. Under the Act, organizations of a political nature cannot accept the Foreign contribution.
3. Political party and publishers of a registered Newspaper can accept Foreign Contribution.

Which of the statements given above is/are correct?


A. 2 only
B. 2 and 3 only
C. 1 only
D. 1, 2 and 3

Correct Answer : A

Answer Justification :

Foreign Contribution (Regulation) Act (FCRA), 2010:

• Foreign funding of voluntary organizations in India is regulated under FCRA act and
is implemented by the Ministry of Home Affairs. Hence, statement 1 is incorrect.

• Under the Act, organizations require to register themselves every five years.

• As per the amended FCRA rules, all NGOs registered or granted prior permission under
FCRA are now required to upload details of foreign contributions received and utilized by
them every three months on their website or the FCRA website.

• NGOs now need to file their annual returns online, with the hard copy version dispensed
with. The annual returns must be placed quarterly on the NGO’s website or the FCRA website
maintained by the home ministry.

Who can accept Foreign Contribution?

A person having a definite cultural, economic, educational, religious or social programme can
accept foreign contribution after getting registration or prior permission from the Central
Government.

Who cannot accept Foreign Contribution?

• Election candidate

www.insightsactivelearn.com 12
Total Marks : 200
Test-2 (Subject)
( INSTA Prelims Test Series 2021 )

• Member of any legislature (MP and MLAs)

• Political party or office bearer thereof

• Organization of a political nature. Hence, statement 2 is correct.

• Correspondent, columnist, cartoonist, editor, owner, printer or publishers of a


registered Newspaper. Hence, statement 3 is incorrect.

• Judge, government servant or employee of any corporation or any other body controlled on
owned by the Government.

• Association or company engaged in the production or broadcast of audio news, audio visual
news or current affairs programmes through any electronic mode

• Any other individuals or associations who have been specifically prohibited by the Central
Government

What is the eligibility criteria for grant of registration?

The Association:

• Must be registered (under the Societies Registration Act, 1860 or Indian Trusts Act 1882 or
section 8 of Companies Act, 2013 etc.)

• Normally be in existence for at least 3 years.

• Has undertaken reasonable activity in its field for the benefit of the society.

• Has spent at least Rs.10,00,000/- (Rs. ten lakh) over the last three years on its activities.

17. Consider the following statements regarding Indian Circus


1. The Great Indian circus was the first modern Indian circus, founded by Vishnupant Chatre.
2. Government of India enforced the ban on the use wild animals for the purpose of
entertainment in Circus.

Which of the statements given above is/are correct?


A. 1 only
B. 2 only
C. Both 1 and 2
D. Neither 1 nor 2

Correct Answer : C

www.insightsactivelearn.com 13
Total Marks : 200
Test-2 (Subject)
( INSTA Prelims Test Series 2021 )

Answer Justification :

Both the statements are correct.

The Great Indian circus was the first modern Indian circus, founded by Vishnupant
Chatre, an accomplished equestrian and singer. It flourished under the patronage of
the Raja of Kurduwadi, under whom Chatre worked as a stable master.

The first performance of The Great Indian Circus was held on March 20, 1880, following which
it toured extensively throughout the country as well as in foreign shores such as Ceylon and
South East Asia. He gained appreciation everywhere. However, his tour to North America was
a failure where he couldn’t match the size and grandiose of his competitors.

Government of India enforced a ban on the use of wild animals for the purpose of
entertainment in 1997, a typical genre of performance and attraction for the
audience hence ceased to exist.

18. The predecessor of Buddha under Buddhism was

A. Kassapa Buddha
B. Maitreya
C. Avalokitāśvara
D. Vajrapani

Correct Answer : A

Answer Justification :

Buddha attained Mahaparinirvana at Kushinagar (Malla Mahajanapada) in Uttar Pradesh at


the age of 80 in 483 BC. He is said to be contemporary for major part of his life to King
Bimbisara and for last few years to Ajatashatru of Haryanka Dynasty.

Buddha is known in various Buddhist texts also as Tathagata and Sakyamuni. The
predecessor of Buddha under Buddhism was Kassapa Buddha and his successor will be
Maitreya. Hence, option (a) is correct.

19. Consider the following statements regarding Khajuraho School of architecture


1. In these temples, only the exterior walls were lavishly decorated with carvings.
2. The temples were made of sandstone.
3. The temples belong to Hindu as well as Jain religion.

Which of the statements given above is/are correct?


A. 1 and 2 only

www.insightsactivelearn.com 14
Total Marks : 200
Test-2 (Subject)
( INSTA Prelims Test Series 2021 )

B. 1 and 3 only
C. 2 and 3 only
D. 1, 2 and 3

Correct Answer : C

Answer Justification :

Khajuraho School:

In the central part of India, the Chandela rulers developed a distinct style of temple making
of their own – known as Khajuraho school or Chandel school.

The features of the temples here include:

In these temples, both the interior and exterior walls were lavishly decorated with carvings.
Hence, statement 1 is incorrect.

The sculptures were generally erotic in their themes and drew inspiration from
Vatsyayana’s Kamasutra.
The temples were made of sandstone.
The temples had three chambers – garbhagriha, mandapa and ardha-mandapa. Some
temples had a vestibular entrance to the garbhagriha known as antarala.
The temples were generally north or east facing.
Panchayatan style of temple making was followed. Even the subsidiary shrines had
Rekha prasad shikharas. This created an impression of a mountain range.
The temples were built on relatively high platform and belong to Hindu as well as Jain
religion.

20. Consider the following statements regarding the Rafale fighter jets:
1. It is a single engine, multirole combat aircraft.
2. It has SCALP, the air-to-ground cruise missile with a range over 300 km.
3. The aircraft is fitted with an Onboard Oxygen Generation System (OBOGS).

Which of the statements given above is/are correct?


A. 1 and 2 only
B. 3 only
C. 2 and 3 only
D. 1, 2 and 3

Correct Answer : C

Answer Justification :

Rafale is a twin-engine, canard-delta wing, multirole combat aircraft designed and

www.insightsactivelearn.com 15
Total Marks : 200
Test-2 (Subject)
( INSTA Prelims Test Series 2021 )

built by French company Dassault Aviation.

Hence, statement 1 is incorrect.

Each aircraft has 14 storage stations for weapons. The jets come with one of the most
advanced Meteor air-to-air missiles. The 190-kg missile has a Beyond Visual Range
(BVR) of over 100 km, traveling at a top speed of Mach 4.

The Rafale jets also come with SCALP, the air-to-ground cruise missile with a range
over 300 km. It is a long-range deep strike missile.

Hence, statement 2 is correct.

The aircraft is fitted with an Onboard Oxygen Generation System (OBOGS). This
suppresses the need for liquid oxygen re-filling or ground support for oxygen
production.

Hence, statement 3 is correct.

21. Consider the following statements regarding Putul Nachh


1. The puppets are generally made of wood and are without any joints
2. It is the traditional rod puppet dance of Bengal-Odisha-Assam region.
3. The performance is accompanied by a musical troop.

Which of the statements given above is/are correct?


A. 1 and 3 only
B. 1 and 2 only
C. 2 and 3 only
D. 1, 2 and 3

Correct Answer : C

Answer Justification :

Putul Nachh is the traditional rod puppet dance of Bengal-Odisha-Assam region. The
figures are generally 3-4 ft. high and dressed like characters of Jatra. Hence Statement 2 is
correct.

They generally have three joints – at the neck and at the shoulders. Hence Statement 1 is
incorrect.

The puppeteers, each of whom control a single puppet by means of a rod attached to his waist,
are placed behind a high curtain. The puppeteers move around behind the curtain, imparting
similar movements to the puppets. The performance is accompanied by a musical troop

www.insightsactivelearn.com 16
Total Marks : 200
Test-2 (Subject)
( INSTA Prelims Test Series 2021 )

of 3-4 musicians playing harmonium, cymbals and tabla. Hence Statement 3 is


correct.

22. Which of the following are the sub-sects under Buddhism?


1. Mahisasaka
2. Gokulika
3. Hemavatika
4. Siddhatthaka

Select the correct answer using the code given below:


A. 1, 2 and 3 only
B. 2, 3 and 4 only
C. 1, 2 and 4 only
D. 1, 2, 3 and 4

Correct Answer : D

Answer Justification :

All the above are correct.

EARLY BUDDHIST SCHOOLS

After passing of Buddha, The original Sangha is believed to have

split into the two early schools in between 383 BC and 250 BC:

Sthavira nikaya (sub-sects included Mahisasaka, Sarvastivada, Sankrantika, Sautrantika,


Dharmaguptaka, Vatsiputriya, Dharmottariya, Bhadrayaniya, Sannagarika and Saṃmitiya)

Mahasaṃghika (sub-sects included Gokulika, Prajnaptivada, Bahusrutiya, Ekavyaharikas,


Caitika)

Other sub-sects not identified under above two schools were Hemavatika, Rajagiriya,
Siddhatthaka, Pubbaseliya, Aparaseliya, and Apararajagirika.

Later Buddhist schools included Theravada, Mahayana, Vajrayana,

etc.

23. Consider the following statements regarding Vesara School of architecture


1. It combined the features of both Nagara School and Dravidian school.

www.insightsactivelearn.com 17
Total Marks : 200
Test-2 (Subject)
( INSTA Prelims Test Series 2021 )

2. Influence of Nagara style is in Curvilinear Shikhara and square base of Vesara temples.
3. Influence of Dravida style is seen in intricate carvings and sculptures, design of Vimana and
Step or terraced Shikara of Vesara temples.

Which of the statements given above is/are correct?


A. 1 only
B. 1 and 3 only
C. 2 and 3 only
D. 1, 2 and 3

Correct Answer : D

Answer Justification :

Vesara School:

Also known as the Karnataka school of architecture, it was conceptualized under the later
Chalukya rulers in the mid-seventh century A.D. It combined features of both Nagara school
and Dravidian school and resulted in a hybridised style. Some of its features are:

1. Emphasis on vimana and mandapa


2. Open ambulatory passageway
3. The pillars, doorways and the ceilings were decorated with intricate carvings.

Three prominent dynasties who made Vesara style temples are:

1. Chalukyas of Badami and Kalyani.


2. Rashtrakutas (750-983 AD). For Example, Kailashnath temple in Ellora, etc.
3. Hoysala Dynasty (1050-1300 AD). For example, temples at Halebid, Belur etc.
Influence of Nagara style is in Curvilinear Shikhara and square base of Vesara temples.
Influence of Dravida style is seen in intricate carvings and sculptures, design of Vimana
and Step or terraced Shikara of Vesara temples.

Example: Doddabasappa temple at Dambal, Ladkhan temple at Aihole, temples at Badami etc.

Hence, option (d) is correct.

24. Consider the following statements regarding the National Education Policy 2020:
1. It aims for Universalization of Education from pre-school to secondary level with 100 % GER in
school education by 2045.
2. Higher Education Commission of India (HECI) will be set up as a single overarching umbrella
body for the entire higher education, including medical and legal education.
3. The policy has emphasized mother tongue/local language/regional language as the medium of
instruction at least till Grade 5.

www.insightsactivelearn.com 18
Total Marks : 200
Test-2 (Subject)
( INSTA Prelims Test Series 2021 )

Which of the statements given above is/are correct?


A. 1 and 2 only
B. 3 only
C. 1 and 3 only
D. 2 only

Correct Answer : B

Answer Justification :

The MHRD initiated an unprecedented collaborative, inclusive, and highly participatory


consultation process from January 2015. In May 2016, ‘Committee for Evolution of the New
Education Policy’ under the Chairmanship of Late Shri T.S.R. Subramanian, Former Cabinet
Secretary, submitted its report. Based on this, the Ministry prepared ‘Some Inputs for the
Draft National Education Policy, 2016’. In June 2017 a ‘Committee for the Draft National
Education Policy’ was constituted under the Chairmanship of eminent scientist Dr. K.
Kasturirangan, which submitted the Draft National Education Policy, 2019 on 31st
May, 2019.

Built on the foundational pillars of Access, Equity, Quality, Affordability and Accountability,
this policy is aligned to the 2030 Agenda for Sustainable Development. It aims for
Universalization of Education from pre-school to secondary level with 100 % GER in
school education by 2030. Hence, statement 1 is incorrect.

The policy has emphasized mother tongue/local language/regional language as the


medium of instruction at least till Grade 5, but preferably till Grade 8 and beyond.
Sanskrit to be offered at all levels of school and higher education as an option for students,
including in the three-language formula. Other classical languages and literatures of India also
to be made available as options. No language will be imposed on any student. Hence,
statement 3 is correct.

Higher Education Commission of India (HECI) will be set up as a single overarching


umbrella body for the entire higher education, excluding medical and legal
education. HECI to have four independent verticals - National Higher Education
Regulatory Council (NHERC) for regulation, General Education Council (GEC ) for
standard setting, Higher Education Grants Council (HEGC) for funding, and National
Accreditation Council( NAC) for accreditation. HECI will function through faceless
intervention through technology, & will have powers to penalise Higher Education Institutions
not conforming to norms and standards. Public and private higher education institutions will
be governed by the same set of norms for regulation, accreditation and academic standards.
Hence, statement 2 is incorrect.

www.insightsactivelearn.com 19
Total Marks : 200
Test-2 (Subject)
( INSTA Prelims Test Series 2021 )

https://pib.gov.in/PressReleasePage.aspx?PRID=1642051

25. Consider the following statements regarding Shadow Puppets


1. Shadow puppets are three dimensional figures cut out of leather.
2. The puppets are placed on a white screen with light falling from behind, creating a shadow on
the screen.
3. Ravanchhaya is theatrical shadow puppetry in the Odisha region.

Which of the statements given above is/are correct?


A. 1 only
B. 1, 2 and 3
C. 1 and 2 only
D. 2 and 3 only

Correct Answer : D

Answer Justification :

India has a rich tradition in shadow puppetry, which has survived till now. Some of the
features of shadow puppetry are:

Shadow puppets are flat figures cut out of leather. Hence Statement 1 is
incorrect.
The figures are painted identically on both sides of the leather.
The puppets are placed on a white screen with light falling from behind,
creating a shadow on the screen. Hence Statement 2 is correct.
The figures are manipulated so that the silhouettes created on the blank screen create
telling imagery

Ravanchhaya is the most theatrical of shadow puppetry and is a popular form of


entertainment in the Odisha region. The puppets are made of deer skin and depict bold,
dramatic postures. They do not have any joints attached to them, making it a more complex
art. Hence Statement 3 is correct.

26. Consider the following statements regarding Mahayana Buddhism


1. It means the greater vehicle.
2. The Mahayana scholars predominantly used Sanskrit as a language.
3. Emperor Ashoka patronised Mahayana school.

Emperor Ashoka patronised Mahayana school.


A. 1 and 2 only
B. 1 and 3 only
C. 2 and 3 only

www.insightsactivelearn.com 20
Total Marks : 200
Test-2 (Subject)
( INSTA Prelims Test Series 2021 )

D. 1, 2 and 3

Correct Answer : A

Answer Justification :

Mahayana Buddhism:

It means the greater vehicle.


The school is more liberal and believes in the heavenliness of Buddha and Bodhisattvas
embodying Buddha Nature. The ultimate goal under Mahayana is “spiritual upliftment”.
The Mahayana followers believe in idol or image worship of Buddha.
The concept of Bodhisattva is the result of Mahayana Buddhism. Mahayana is also
called “Bodhisattvayana”, or the “Bodhisattva Vehicle. That is to say, the followers
believe in Bodhisattva concept of salvation of all conscious individual.
In other words, they believe in universal liberation from suffering of all beings.
A bodhisattva seeks complete enlightenment for the benefit of all beings. A bodhisattva
who has accomplished this goal is called a Samyaksaṃbuddha.
Prominent Mahayana texts include Lotus Sutra, Mahavamsa, etc.
As per Lotus Sutra, Mahayana school believes in six perfections (or paramitas) to be
followed by an individual.
The Mahayana scholars predominantly used Sanskrit as a language.

Emperor Ashoka patronised Hinayana sect as Mahayana school came into being much later.
Hence, statement 3 is incorrect.

27. Consider the following statements regarding temples developed during the Pallava dynasty
1. The temples built under Narasimhavarman were basically rock-cut temples.
2. Under Mahendravarman, development of real structural temples was started in the place of
rock-cut temples.

Which of the statements given above is/are correct?


A. 1 only
B. 2 only
C. Both 1 and 2
D. Neither 1 nor 2

Correct Answer : D

Answer Justification :

The temples developed during the Pallava dynasty reflected the stylistic taste of the individual
rulers and can be classified into four stages chronologically.

www.insightsactivelearn.com 21
Total Marks : 200
Test-2 (Subject)
( INSTA Prelims Test Series 2021 )

Mahendra group:

This was the first stage of Pallava temple architecture. The temples built under
Mahendravarman were basically rock-cut temples. Under him, the temples were known as
mandapas, unlike the Nagara style in which the mandapas meant only the assembly hall.
Hence, statement 1 is incorrect.

Narasimha group:

This represented the second stage of the development of temple architecture in South India.
The rock-cut temples were decorated by intricate sculptures. Under Narasimhavarman, the
mandapas were now divided into separate rathas. The biggest one was called the Dharmaraja
ratha while the smallest one was called the Draupadi ratha. The design of a temple in the
Dravidian style of architecture is a successor of the Dharmaraja ratha.

Rajsimha group:

Rajsimhavarman led the third stage of temple development. Under him, development of real
structural temples was started in the place of rock-cut

temples. Hence, statement 2 is incorrect.

Example: Shore temple at Mahabalipuram, Kailashnath temple at Kanchipuram, etc.

Nandivarman group:

This was the fourth stage of temple development during the Pallava dynasty. The temples built
under him were smaller in size. The features were almost similar to the Dravidian style of
temple architecture. After the decline of Pallava dynasty, temple architecture acquired a new
style under the Chola kingdom, known as the Dravidian style of temple architecture. This
marked a new era in the development of temples in the Southern India.

28. Consider the following statements regarding the National Viral Hepatitis Control Program (NVHCP):
1. It is a comprehensive plan covering the entire gamut from Hepatitis A, B, C, D and E.
2. It aims to combat hepatitis and achieve country wide elimination of Hepatitis C by 2030.

Which of the statements given above is/are correct?


A. 1 only
B. 2 only
C. Both 1 and 2
D. Neither 1 nor 2

www.insightsactivelearn.com 22
Total Marks : 200
Test-2 (Subject)
( INSTA Prelims Test Series 2021 )

Correct Answer : C

Answer Justification :

Both the statements given above are correct.

The National Viral Hepatitis Control Program has been launched by Ministry of
Health and Family Welfare, Government of India on the occasion of the World
Hepatitis Day, 28th July 2018. It is an integrated initiative for the prevention and
control of viral hepatitis in India to achieve Sustainable Development Goal (SDG) 3.3
which aims to ending viral hepatitis by 2030. This is a comprehensive plan covering
the entire gamut from Hepatitis A, B, C, D & E, and the whole range from prevention,
detection and treatment and also mapping treatment to outcomes. Operational
Guidelines for National Viral Hepatitis Control Program, National Laboratory Guidelines for
Viral Hepatitis Testing and National Guidelines for Diagnosis and Management of Viral
Hepatitis were also released.

Aims of the Program:

To Combat hepatitis and achieve country wide elimination of Hepatitis C by


2030;
Achieve significant reduction in the infected population, morbidity and mortality
associated with Hepatitis B and C viz. Cirrhosis and Hepato-cellular carcinoma (liver
cancer);
Reduce the risk, morbidity and mortality due to Hepatitis A and E.

https://www.nhp.gov.in/national-viral-hepatitis-control-program-(nvhcp)_pg

https://www.who.int/campaigns/world-hepatitis-day/2020

29. Which of the following dance form uses speech during its performance?

A. Manipuri Dance
B. Sattriya
C. Odissi Dance
D. Kuchipudi

Correct Answer : D

Answer Justification :

Kuchipudi is originally from Andhra Pradesh. Its evolution can be traced to traditional

www.insightsactivelearn.com 23
Total Marks : 200
Test-2 (Subject)
( INSTA Prelims Test Series 2021 )

dance - drama, known under the generic name of Yakshagaana.

The dance is accompanied by song which is typically Carnatic music. Like other classical
dances, Kuchipudi also comprises pure dance, mime and histrionics but it is the use of
speech that distinguishes Kuchipudi's presentation as dance drama.

30. Consider the following statements regarding auspicious symbols under Jainsim
1. Swastika: It signifies peace and wellbeing of the humans.
2. Nandyavartya: The mirror which reflects the inner self.
3. Bhadrasana: A throne which is said to be sancrified by the jaina’s feet.

Which of the statements given above is/are correct?


A. 1 and 2 only
B. 1 and 3 only
C. 2 and 3 only
D. 1, 2 and 3

Correct Answer : B

Answer Justification :

Sectarian lists the following eight auspicious symbols under Jainsim:

Swastika: It signifies peace and wellbeing of the humans.

Nandyavartya: It is a large swastika with nine end points. Hence, statement 2 is incorrect.

Bhadrasana: A throne which is said to be sancrified by the jaina’s feet.


Shrivasta: A mark which manifested on the Jain’s chest and signified his pure soul.
Darpana: The mirror which reflects the inner self
Minayugala: A couple of fish which signifies the conquest over sexual urges
Vardhamanaka: A shallow dish used as lamp which shows the increase in the wealth,
due and merit.
Kalasha: A pot filled with pure water signifying water.

31. Consider the following statements regarding Mandu architecture


1. Mandu is a fine example of architectural adaptation to the environment.
2. The Hindola Mahal looks like a railway viaduct bridge with its disproportionately large
buttresses supporting the walls.
3. Mandu was the capital city of Ghauri Dynasty.

Which of the statements given above is/are correct?


A. 1 only
B. 1 and 3 only

www.insightsactivelearn.com 24
Total Marks : 200
Test-2 (Subject)
( INSTA Prelims Test Series 2021 )

C. 2 and 3 only
D. 1, 2 and 3

Correct Answer : D

Answer Justification :

The city of Mandu is located sixty miles from Indore, at an elevation of over 2000 feet and
overlooks the Malwa Plateau to the north and the Narmada valley to the south. Mandu’s
natural defence encouraged consistent habitations by Parmara Rajputs, Afghans and Mughals.
As the capital city of Ghauri Dynasty (1401–1561) founded by Hoshang Shah it
acquired a lot of fame. Subsequently, Mandu was associated with the romance of Sultan Baz
Bahadur and Rani Rupmati. The Mughals resorted to it for pleasure during the monsoon
season. Mandu is a typical representation of the medieval provincial style of art and
architecture. It was a complex mix of official and residential-cum-pleasure palace, pavilions,
mosques, artificial reservoirs, baolis, embattlements, etc. In spite of the size or monumentality,
the structures were very close to nature, designed in the style of arched pavilions, light and
airy, so that these buildings did not retain heat. Local stone and marble were used to great
advantage. Mandu is a fine example of architectural adaptation to the environment.

The Royal Enclave located in the city comprised the most complete and romantic set of
buildings, a cluster of palaces and attendant structures, official and residential, built around
two artificial lakes.

The Hindola Mahal looks like a railway viaduct bridge with its disproportionately
large buttresses supporting the walls. This was the audience hall of the Sultan and also the
place where he showed himself to his subjects. Batter was used very effectively to give an
impression of swinging (Hindola) walls.

Jahaaz Mahal is an elegant two-storey ‘shippalace’ between two reservoirs, with open
pavilions, balconies overhanging the water and a terrace. Built by Sultan Ghiyasuddin Khilji it
was possibly used as his harem and as the ultimate pleasure and recreational resort. It had a
complex arrangement of watercourses and a terrace swimming pool.

Rani Rupmati’s double pavilion perched on the southern embattlements afforded a beautiful
view of the Narmada valley. Baz Bahadur’s palace had a wide courtyard ringed with halls and
terraces.

A madrasa called Asharfi Mahal now lies in ruins. Hoshang Shah’s tomb is a majestic structure
with a beautiful dome, marble jali work, porticos, courts and towers. It is regarded as an
example of the robustness of Afghan structures, but its lattice work, carved brackets and
toranas lend it a softer hue.

www.insightsactivelearn.com 25
Total Marks : 200
Test-2 (Subject)
( INSTA Prelims Test Series 2021 )

32. Consider the following statements regarding the Global Forest Resources Assessment (FRA), 2020:
1. It has been released out by the Food and Agriculture Organization (FAO) of the United
Nations.
2. During the last decade (2010-20), India has shown the decline in forest cover.
3. Brazil reported the maximum employment in the forestry sector in the world.

Which of the statements given above is/are correct?


A. 1 and 2 only
B. 1 only
C. 3 only
D. None

Correct Answer : B

Answer Justification :

Global Forest Resources Assessment 2020 has been released by the United Nations
Food and Agriculture Organization (FAO). Hence, statement 1 is correct.

FAO has brought out this comprehensive assessment every five years since 1990. This
report assesses the state of forests, their conditions and management for all member
countries.

The FRA 2020 has examined the status of, and trends in, more than 60 forest-related
variables in 236 countries and territories in the period 1990–2020.

During the decade under assessment, India reported 0.38 per cent annual gain in
forest, or 266,000 ha of forest increase every year at an average. The FRA 2020 has
credited the government’s Joint Forest Management programme for the significant increase in
community-managed forest areas in the Asian continent. Hence, statement 2 is incorrect.

The assessment examined employment in the forestry sector (including logging) with
data from 136 countries that represent 91 per cent of the world’s forests. India
reported the maximum employment in the forestry sector in the world.

Globally, 12.5 million people were employed in the forestry sector. Out of this, India accounted
for 6.23 million, or nearly 50 per cent. Hence, statement 3 is incorrect.

https://www.downtoearth.org.in/news/forests/india-among-top-10-countries-gaining-forest-area
-in-the-world-fao-72454

33. Consider the following statements regarding Kathak Dance


1. An important feature of Kathak is the development of different gharanas.
2. Tarana is a special feature in which the dancer recites complicated bols and demonstrates
them
www.insightsactivelearn.com 26
Total Marks : 200
Test-2 (Subject)
( INSTA Prelims Test Series 2021 )

Which of the statements given above is/are correct?


A. 1 only
B. 2 only
C. Both 1 and 2
D. Neither 1 nor 2

Correct Answer : A

Answer Justification :

Tracing its origins from the Ras Leela of Brajbhoomi, Kathak is the traditional dance form of
Uttar Pradesh. Kathak derived its name from the ‘Kathika’ or the story-tellers who recited
verses from the epics, with gestures and music.

An important feature of Kathak is the development of different gharanas as it is


based on Hindustani style of music:

Lucknow: Reached its peak under the reign of Nawab Wajid Ali Khan. It puts more
importance on expression and grace.

Jaipur: Initiated by Bhanuji, it emphasised fluency, speed and long rhythmic patterns.

Raigarh: It developed under the patronage of Raja Chakradhar Singh. It is unique in its
emphasis on percussion music.

Jugalbandi is the main attraction of kathak recital which shows a competitive play between the
dancer and the table player.

Padhant is a special feature in which the dancer recites complicated bols and
demonstrates them. Hence Statement 2 is incorrect.

34. Which of the following are the sub-sects under Digambaras


1. Bispanthi
2. Terapanthi
3. Sthanakavasi

Which of the statements given above is/are correct?


A. 1 and 2 only
B. 1 and 3 only
C. 2 and 3 only
D. 1, 2 and 3

Correct Answer : A

www.insightsactivelearn.com 27
Total Marks : 200
Test-2 (Subject)
( INSTA Prelims Test Series 2021 )

Answer Justification :

Jainism has two major ancient sub traditions:

Digambaras [Sub-sects include Mula Sangh (original community) and Terapanthi, Taranpathi
and Bispanthi (these three are modern communities)]

Svetambaras (Sub-sects include Sthanakavasi and Murtipujaka)

There are several other smaller sub-traditions that emerged in the 2nd millennium CE. Hence,
statement 3 is incorrect.

35. Consider the following statements regarding Indo-Islamic architecture


1. The unique feature was the use of foreshortening technique in the buildings.
2. Pietra-dura technique was the use of geometrical vegetal ornamentation
3. Arabesque method was used for inlay of precious stones and gems into the stone walls.

Which of the statements given above is/are correct?


A. 1 only
B. 1 and 3 only
C. 2 and 3 only
D. 1, 2 and 3

Correct Answer : A

Answer Justification :

Arabesque method was used for decoration. Arabesque meant the use of geometrical vegetal
ornamentation and was characterised by a continuous stem which split regularly, producing a
series of counterpoised, leafy, secondary stems. The secondary stems, in turn, split again or
returned to merge with the main stem creating a decorative pattern. Hence, statement 2 is
incorrect.

The Islamic rulers introduced the Charbagh style of gardening, in which a square block was
divided into four adjacent identical gardens.

The architectures of these times also used the pietra-dura technique for the inlay of precious
stones and gems into the stone walls. Hence, statement 3 is incorrect.

Another unique feature was the use of foreshortening technique in the buildings, such that
the inscriptions appear to be closer than it really is.

36. Which of the following strait/canal connects the Black Sea and Sea of Marmara?

www.insightsactivelearn.com 28
Total Marks : 200
Test-2 (Subject)
( INSTA Prelims Test Series 2021 )

A. Strait of Gibraltar
B. Strait of the Dardanelles
C. Strait of Bosporus
D. Suez Canal.

Correct Answer : C

Answer Justification :

The Bosporus Strait connects the Black Sea to the Sea of Marmara. It also separates
Asian Turkey from European Turkey. Hence, it separates the two continents.

Extra Learning:

Strait of Gibraltar is a channel which connects the Mediterranean Sea with the
Atlantic Ocean.

www.insightsactivelearn.com 29
Total Marks : 200
Test-2 (Subject)
( INSTA Prelims Test Series 2021 )

The Dardanelles Strait is a narrow channel of water that connects the Aegean Sea to the
Sea of Marmara.

The Suez Canal is an artificial sea-level waterway in Egypt, connecting the Mediterranean
Sea to the Red Sea through the Isthmus of Suez.

www.insightsactivelearn.com 30
Total Marks : 200
Test-2 (Subject)
( INSTA Prelims Test Series 2021 )

37. In which of the following dance forms does Lord Sri Krishna is a principal figure?
1. Sankirtana
2. Bharatanatyam
3. Manipuri Dance

Select the correct answer using the code given below


A. 1 and 3 only
B. 2 only
C. 2 and 3 only
D. 1 only

Correct Answer : A

Answer Justification :

Manipuri dance form finds its mythological origin to the celestial dance of Shiva and Parvati
in the valleys of Manipur along with the local ‘Gandharvas’. The dance gained prominence
with the advent of Vaishnavism in 15th century. Then, Krishna became the central theme
of this dance form. It is performed generally by females

Sankirtana is a ritual singing, drumming and dancing art form of Manipur. This art
form is performed to mark religious occasions and various stages in the life of Manipuri
Vaishnavites. Practiced at temples, the performers here narrate the life and deeds of
Lord Krishna through songs and dances.

Bharatnatyam has Lord Shiva as principal figure.

www.insightsactivelearn.com 31
Total Marks : 200
Test-2 (Subject)
( INSTA Prelims Test Series 2021 )

38. Consider the following statements regarding Pratikraman under Jainsim


1. It is a religious practice of voluntarily fasting to death.
2. It is a process during which Jains repent for their sins during their daily life.

Which of the statements given above is/are correct?


A. 1 only
B. 2 only
C. Both 1 and 2
D. Neither 1 nor 2

Correct Answer : B

Answer Justification :

POPULAR PRACTICES UNDER JAINISM

Sallekhana – It is a religious practice of voluntarily fasting to death by gradually reducing


the intake of food and liquids. It is not considered as a suicide by Jain scholars because it
is not an act of passion, nor does it deploy poisons or weapons. It can be performed by both
Jain ascetics and householders. Hence, statement 1 is incorrect.

In 2015, the Rajasthan High Court banned the practice, considering it suicide. Later that year,
the Supreme Court of India stayed the decision of the Rajasthan High Court and lifted the ban
on Sallekhana.

Pratikraman – It is a process during which Jains repent for their sins during their daily
life, and remind themselves not to repeat them. The five types of Pratikaman include Devasi,
Rayi, Pakhi, Chaumasi and Samvatsari.

39. Consider the following statements regarding European architecture


1. The French brought with them the Iberian style of architecture.
2. The Portuguese introduced the concept of Patio Houses.

Which of the statements given above is/are correct?


A. 1 only
B. 2 only
C. Both 1 and 2
D. Neither 1 nor 2

Correct Answer : B

www.insightsactivelearn.com 32
Total Marks : 200
Test-2 (Subject)
( INSTA Prelims Test Series 2021 )

Answer Justification :

Portugese Influence

The Portuguese brought with them the Iberian style of architecture. Hence, statement 1 is
incorrect.

They initially built trading posts and ware houses, which were later remodelled into fortified
town along the coastlines. They also introduced the concept of ‘patio houses’ and ‘Baroque
style’ developed in late-16th century in Europe to express the strength of the Church. It had
elaborate, detailed and theatrical design to create adramatic effect. It involved use of
contrasting colours.

French influence

The French brought the concept of urban city planning with them. The French towns of
Pondicherry and Chandernagore (now Chandannagar, West Bengal) were built using
the Cartesian grid plans and scientific architectural designs. They built imposing buildings as a
show of power. They also introduced the concept of anonymous architecture which involves
simple facade without much ornamentation or design, much like modern buildings.

40. Consider the following pairs of religious festival and their significance:
Festivals: Significance:
1. Losar Birth anniversary of Guru Rinpoche
2. Jamshedi Navroz Festival of New Year for the Parsi Community.
3. Hemis Festival New Year festival in Tibetan Buddhism.

Which of the following pairs is/are correctly matched?


A. 1 and 3 only
B. 2 only
C. 1 and 2 only
D. 2 and 3 only

Correct Answer : B

Answer Justification :

Losar is the first day of the first month of the Tibetan lunar calendar. It is celebrated
on various dates depending on location (Tibet, Bhutan, Nepal, and India) tradition.
Hence, pair 1 is incorrectly matched.

www.insightsactivelearn.com 33
Total Marks : 200
Test-2 (Subject)
( INSTA Prelims Test Series 2021 )

The Parsi New Year, also known as Navroz or Jamshedi Navroz, is celebrated every year to
mark the beginning of the new Iranian calendar. Hence, pair 2 is correctly matched.

Hemis Festival. It is dedicated to Lord Padmasambhava or popularly known as Guru


Rinpoche. The Dance Festival is world famous representing the reincarnation of
Buddha. Hence, pair 3 is incorrectly matched.

41. Consider the following pairs


Dance Form Practiced in
1. Kolkali-Parichakali Lakshadwee
2. Bihu Assam
3. Kaikottikali Karnataka

Which of the pairs given above is/are matched correctly?


A. 1 and 3 only
B. 1 and 2 only
C. 2 and 3 only
D. 2 only

Correct Answer : B

Answer Justification :

Kolkali-Parichakali is a popular martial dance in the areas of southern Kerala and


Lakshwadeep. Kol means stick and Paricha means shield. The dancers use mock weapons
made of wood and enact fight sequences.

Bihu is the popular dance of Assam, performed in group by both men and women. The
dancers are dressed in colourful traditional dresses to celebrate the pomp and gaiety. The
dance performance includes group formations, rapid hand movements and brisk footsteps.

Kaikottikali is a popular temple dance of Kerala. It is performed by both men and women
at the time of Onam to celebrate the rich harvest. Airukali and Tattamakali are similar forms of
this dance.

42. Consider the following statements regarding Zoroastrianism


1. This religion has its origin in Persia by the prophet Zarathustra around 6-7 AD.
2. They are a monotheistic religion who believe in one eternal god whose name is Ahura Mazda.
3. They worship fire but also consider air, water and earth to be sacred elements.

Which of the statements given above is/are correct?


A. 1 only

www.insightsactivelearn.com 34
Total Marks : 200
Test-2 (Subject)
( INSTA Prelims Test Series 2021 )

B. 1 and 3 only
C. 2 and 3 only
D. 1, 2 and 3

Correct Answer : C

Answer Justification :

Zoroastrianism

This religion has its origin in Persia by the prophet Zarathustra around 6-7 BCE. Hence,
statement 1 is incorrect.

They are a monotheistic religion which believes in one eternal god whose name is Ahura
Mazda who is the epitome of just behaviour and goodness. There is a spirit of malice and bad
behaviour called Angra Mainyu. These two-keep fighting with each other and this is an
eternal struggle and one day good will win over evil and that will be the last day.

They are commonly known as Parsis and are presently amongst the smallest (and rapidly
shrinking) communities in India. They live mostly in Mumbai, Goa and Ahmedabad. Their fire
temples called Atash Bahram are rare and only eight known temples exist in the entire
country.

Their sacred text called Zend Avesta is written in Old Avestan.

They worship fire but also consider air, water and earth to be sacred elements.

43. Which of the following Buddhist texts was/were written in Sanskrit language?
1. Mahavastu
2. Lalitavistara Sutra
3. Buddhacharita

Select the correct answer using the code given below:


A. 1 and 2 only
B. 2 and 3 only
C. 1 and 3 only
D. 1, 2 and 3

Correct Answer : D

Answer Justification :

All the above given texts were written in Sanskrit language.

www.insightsactivelearn.com 35
Total Marks : 200
Test-2 (Subject)
( INSTA Prelims Test Series 2021 )

The Mahavastu is a text of the Lokottaravada school of Early Buddhism. Mora than half
of the text is composed of Jataka and Avadana tales, accounts of the earlier lives of the Buddha
and other bodhisattvas.

The Mahavastu contains prose and verse written in mixed Sanskrit, Pali and Prakrit.

The Lalitavistara Sutra is a Sanskrit Mahayana Buddhist sutra that tells the story of
Gautama Buddha from the time of his descent from Tushita until his first sermon in the Deer
Park near Varanasi.

Buddhacharita is an epic poem in the Sanskrit mahakavya style on the life of Gautama
Buddha by Asvaghoṣa, composed in the 2nd century A.D.

44. Consider the following pairs


Tribes Largely located in
1.Mising tribe Assam
2. Todas Telangana
3. Bru Tribes Sikkim

Which of the pairs given above is/are matched correctly?


A. 1 and 2 only
B. 3 only
C. 1 and 3 only
D. 1 only

Correct Answer : D

Answer Justification :

Misings are an indigenous community inhabiting parts of the Indian states of Assam
and Arunachal Pradesh. They were also known as Miris in the past and still recognized as
Miris in the Constitution of India. Hence statement 1 is correctly matched.

Toda people are a Dravidian ethnic group who live in the Nilgiri Mountains of the
Indian state of Tamil Nadu. Before the 18th century and British colonisation, the Toda
coexisted locally with other ethnic communities, including the Kota, Badaga and Kurumba, in a
loose caste-like society, in which the Toda were the top ranking. During the 20th century, the
Toda population has hovered in the range 700 to 900. Hence statement 2 is incorrectly
matched.

Reang (aka Bru in Mizoram) are one of the 21 scheduled tribes of the Indian state of

www.insightsactivelearn.com 36
Total Marks : 200
Test-2 (Subject)
( INSTA Prelims Test Series 2021 )

Tripura. The Bru can be found all over the Tripura state in India. However, they may
also be found in Mizoram and Assam. They speak the Reang dialect of Kokborok language
which is of Tibeto-Burmese origin and is locally referred to as Kau Bru. Hence statement 3 is
incorrectly matched.

45. Consider the following statements regarding Folk Dances


1. Alkap is mainly performed solo, while balancing a number of earthen pots on the head.
2. The dancers are armed with wooden spears and shields in Paika Dance
3. Gaur Maria imitates the movements of a bison and is performed in group by both men and
women.

Which of the statements given above is/are correct?


A. 1 and 3 only
B. 2 only
C. 1, 2 and 3
D. 2 and 3 only

Correct Answer : D

Answer Justification :

Matki is performed by the women of Malwa region on the occasions of wedding and other
festivities. It is mainly performed solo, while balancing a number of earthen pots on
the head. Hence statement 1 is incorrect.

Paika is a martial folk dance performed in the southern parts of Odisha. Paika is a
form of long spear. The dancers are armed with wooden spears and shields, and show off
their skills and agility in infantry like formations. It has a martial art character. The word
Paika signifies battle. Hence Statement 2 is correct.

Gaur Maria is an important ritualistic dance form of the Bison Horn Maria tribes who
live in the Bastar region of Chhattisgarh. The dance imitates the movements of a bison
and is performed in group by both men and women. Hence Statement 3 is correct.

Paika is a martial folk dance performed in the southern parts of Odisha. Paika is a form of long
spear. The dancers are armed with wooden spears and shields, and show off their skills and
agility in infantry like formations. It has a martial art character. The word Paika signifies
battle.

46. Which of the following places were visited by Gautama Buddha?


1. Kaushambi
2. Shravasti
3. Rajgriha

www.insightsactivelearn.com 37
Total Marks : 200
Test-2 (Subject)
( INSTA Prelims Test Series 2021 )

4. Varanasi

Select the correct answer using the code given below:


A. 1, 2 and 3 only
B. 2, 3 and 4 only
C. 1, 2 and 4 only
D. 1, 2, 3 and 4

Correct Answer : D

Answer Justification :

Gautama Buddha delivered his first sermon at Sarnath in Banaras. He undertook long journeys
and carried his message far and wide. He had a very strong physique, and this enabled him to
walk 20 to 30 km a day. He kept wandering, preaching, and meditating continually for forty
years, resting only during the annual rainy season. During this long period he encountered
many staunch supporters of rival sects, including the brahmanas, but defeated them in
debates. His missionary activities did not discriminate between the rich and the poor, the high
and the low, and man and woman. Gautama Buddha passed away at the age of 80 in 487 BC at
a place called Kusinagara, coterminous with the village called Kasia in Deoria district in
eastern UP. However, as in the case of Vardhamana Mahavira, the existence of Gautama
Buddha in the sixth century BC is not supported by archaeological evidence. The cities
Kaushambi, Shravasti, Varanasi, Vaishali, and Rajgriha, were visited by Buddha.
Rajgriha did not assume any urban character until the 5th BCE though.

Hence, option (d) is correct.

47. Brahui, Malto and Kurukh languages belong to:

A. Indo-Aryan Linguistic Family


B. Dravidian Linguistic Family
C. Sino-Tibetan Linguistic Family
D. Austric Linguistic Family

Correct Answer : B

Answer Justification :

Brahui is a Dravidian language spoken mainly by the Brahui people in the middle region
of Balochistan Province, in Pakistan.

Malto is a Northern Dravidian language spoken primarily in the tribal areas of


Bengal.

www.insightsactivelearn.com 38
Total Marks : 200
Test-2 (Subject)
( INSTA Prelims Test Series 2021 )

Kurukh is a Dravidian language spoken by nearly two million tribal people of


Jharkhand, Madhya Pradesh, Chhattisgarh, Odisha and West Bengal in India, as well
as in northern Bangladesh.

48. Consider the following statements regarding Monoclonal Antibodies


1. Monoclonal antibodies are proteins cloned in the lab to mimic antibodies produced by the
immune system to counter an infection.
2. Monoclonal antibodies are used to treat many diseases, including some types of cancer.

Which of the statements given above is/are correct?


A. 1 only
B. 2 only
C. Both 1 and 2
D. Neither 1 nor 2

Correct Answer : C

Answer Justification :

Both the statements are correct.

Monoclonal antibodies are proteins cloned in the lab to mimic antibodies produced by
the immune system to counter an infection. They have their genesis in serum, the
colourless constituent of blood that contains antibodies.

Monoclonal antibodies are used to treat many diseases, including some types of
cancer. To make a monoclonal antibody, researchers first have to identify the right antigen to
attack. Finding the right antigens for cancer cells is not always easy, and so far mAbs have
proven to be more useful against some cancers than others.

https://www.thehindu.com/sci-tech/health/the-hindu-explains-why-is-a-psoriasis-drug-being-use
d-to-treat-covid-19-patients/article32126359.ece

https://www.cancer.org/treatment/treatments-and-side-effects/treatment-types/immunotherapy
/monoclonal-antibodies.html

49. Which one of the following texts of ancient India allows divorce to a wife who has been deserted by her
husband?

A. Kamasutra
B. Manava Dharma Shastra
C. Sukra Nitisara
D. Arthashastra

www.insightsactivelearn.com 39
Total Marks : 200
Test-2 (Subject)
( INSTA Prelims Test Series 2021 )

Correct Answer : D

Answer Justification :

The Arthashastra is an Indian treatise on politics, economics, military strategy, the


function of the state, and social organization attributed to the philosopher and Prime
Minister Kautilya (also known as Chanakya, Vishnugupta, l. c. 350-275 BCE) who was
instrumental in establishing the reign of the great king Chandragupta Maurya (r. c. 321-c.297
BCE), founder of the Mauryan Empire (322-185 BCE).

The Arthashastra is thought to have been written by Kautilya as a kind of handbook for
Chandragupta instructing him in how to reign over a kingdom and encouraging direct action in
addressing political concerns without regard for ethical considerations.

Arthashastra allows divorce to a wife deserted by her husband

50. Which of the following miniatures are generally designated as the Kulhadar Group
1. Chaurapanchasika
2. Fifty Verses of the Thief by Bilhan
3. Gita Govinda
4. Ragamala

Select the correct answer using the code given below:


A. 1, 2 and 3 only
B. 2, 3 and 4 only
C. 1, 2 and 4 only
D. 1, 2, 3 and 4

Correct Answer : D

Answer Justification :

The finest examples of painting belonging to the first half of the 16th century are,
represented by a group of miniatures generally designated as the "Kulhadar Group".
This group includes illustrations of the 'Chaurapanchasika' - "Fifty Verses of the Thief
by Bilhan, the Gita Govinda, the Bhagavata Purana and Ragamala.

Hence, option (d) is correct.

The style of these miniatures is marked by the use of brilliant contrasting colours, vigorous
and angular drawing, transparent drapery and the appearance of conical caps 'Kulha' on which
turbans are worn by the male figures.

An example of the Chaurapanchasika miniature shows Champavati standing near a lotus pond.

www.insightsactivelearn.com 40
Total Marks : 200
Test-2 (Subject)
( INSTA Prelims Test Series 2021 )

This miniature belongs to the N.C. Mehta collection, Bombay. It was executed in the first
quarter of the 6th century, probably in Mewar. The style of the painting is purely indigenous
derived from the earlier tradition of the Westen Indian art and does not show any influence of
either the Persian or the Mughal style of painting.

51. Consider the following statements regarding the languages in India:


1. Hindi is the national language of India.
2. The official language to be adopted by the States shall be one of those listed in the Eighth
Schedule.
3. Recently, English has been added to the list of scheduled languages under the Eighth
Schedule.

Which of the following statements given is/are correct?


A. 1 and 2 only
B. 3 only
C. 1, 2 and 3
D. None

Correct Answer : D

Answer Justification :

All the statements given above are incorrect.

In the constitution, Hindi was declared as an official language and not a national
language. Under Article 343, official language of the Union has been prescribed,
which includes Hindi in Devanagari script and English.

The Constitution does not specify the official language to be used by the States for
the conduct of official function. States are free to adopt it. The language to be adopted by
the States need not be one of those listed in the Eighth Schedule.

The Eighth Schedule to the Constitution consists of the following 22 languages:- (1)
Assamese, (2) Bengali, (3) Gujarati, (4) Hindi, (5) Kannada, (6) Kashmiri, (7)
Konkani, (8) Malayalam, (9) Manipuri, (10) Marathi, (11) Nepali, (12) Oriya, (13)
Punjabi, (14) Sanskrit, (15) Sindhi, (16) Tamil, (17) Telugu, (18) Urdu (19) Bodo, (20)
Santhali, (21) Maithili and (22) Dogri. Of these languages, 14 were initially included in the
Constitution. Sindhi language was added in 1967. Thereafter three more languages viz.,
Konkani, Manipuri and Nepali were included in 1992. Subsequently Bodo, Dogri,
Maithili and Santhali were added in 2004.

52. Sakteng, sometimes seen in the news, is located in which of the following state?

www.insightsactivelearn.com 41
Total Marks : 200
Test-2 (Subject)
( INSTA Prelims Test Series 2021 )

A. Assam
B. Mizoram
C. Arunachal Pradesh
D. None of the above

Correct Answer : D

Answer Justification :

Sakteng Wildlife Sanctuary is a wildlife sanctuary located mostly in Trashigang


District and just crossing the border into Samdrup Jongkhar District, Bhutan. It is
one of the country's protected areas. It is listed as a tentative site in Bhutan's
Tentative List for UNESCO inclusion.

https://www.thehindu.com/news/international/days-after-demarche-china-doubles-down-on-clai
ms-on-eastern-bhutan-boundary/article31993470.ece

53. Consider the following statements regarding Folk Theatre


1. Kuruvanji, a tamil folk theatre revolves around a love-struck heroine.
2. Naqal is the mimicry based performance based in Punjab.

Which of the statements given above is/are correct?


A. 1 only
B. 2 only
C. Both 1 and 2
D. Neither 1 nor 2

Correct Answer : C

Answer Justification :

Both the statements are correct

Kuruvanji

Originated around 300 years ago, Kuruvanji is characterised by classical Tamil poetry
and songs.

The first Kuruvanji was composed by Thirukutarajappa Kaviyar. The basic theme revolves
around a love-struck heroine. Kuruvanji literally means ‘fortune-teller’ who predicts the
fate of the heroine. Kuruvanji is performed in a dance ballet form with Bharatnatyam being the
principal dance form in Tamil Nadu.

Naqal or Bhand

www.insightsactivelearn.com 42
Total Marks : 200
Test-2 (Subject)
( INSTA Prelims Test Series 2021 )

It is mimicry based performance, based in Punjab and adjoining areas. The


performing artist is often called ‘Behroopiya’ or ‘Naqalchi’ (impersonator). He uses
comedy, absurdities, wit and humour to get across the social or political message. It is
performed by two artists.

54. Match the following Styles of Paintings with their Major features
1. Cauka Pitaka: Isolated framed drawings
2. Dighala Pitaka: Isolated paintings
3. Yama Pitaka: Long scrolls of paintings

Which of the pairs given above is/are correctly matched?


A. 1 only
B. 1 and 3 only
C. 2 and 3 only
D. 1, 2 and 3

Correct Answer : A

Answer Justification :

The play, Mudrarakshasa by Vishakhadutta, also facilitated the reader by mentioning the name
of various paintings or patas, which are important to understand the different style of
paintings and to observe all the principles of paintings. Some of the styles were:

Styles of Paintings Major features

Cauka Pitaka: Isolated framed drawings

Dighala Pitaka: Long scrolls of paintings. Hence, statement 2 is incorrect.

Yama Pitaka: Isolated paintings. Hence, statement 3 is incorrect.

55. Consider the following pairs of literary works and their authors:
Literary Works: Author
1. Panchatantra Narayana Pandit
2. Hitopadesh Vishnu Sharma
3. Ashtadhyayi Panini

Which of the pairs given above is/are correctly matched?


A. 1 and 2 only

www.insightsactivelearn.com 43
Total Marks : 200
Test-2 (Subject)
( INSTA Prelims Test Series 2021 )

B. 2 and 3 only
C. 3 only
D. None

Correct Answer : C

Answer Justification :

The Panchatantra is an ancient Indian collection of interrelated animal fables in


Sanskrit verse and prose, arranged within a frame story. It was written by Vishnu
Sharma. Hence, pair 1 is incorrectly matched.

Hitopadesha is an age-old compilation of stories compiled by Narayana Pandit over a


thousand years ago. It is an Indian text in the Sanskrit language consisting of fables
with both animal and human characters. Hence, pair 2 is incorrectly matched.

Ashtadhyayi is a Sanskrit treatise on grammar written by the Indian grammarian


Panini during the 6th to 5th century BCE. This work set the linguistic standards for
Classical Sanskrit. Hence, pair 3 is correctly matched.

56. Which of the following is/are the part of Quadrilateral Security Dialogue (QSD)?
1. United States of America
2. South Korea
3. China
4. Australia

Select the correct answer using the code given below


A. 2 and 3 only
B. 1 and 4 only
C. 1 only
D. 1, 2 and 4 only

Correct Answer : B

Answer Justification :

Quadrilateral Security Dialogue (Quad) is the informal strategic dialogue between


India, USA, Japan and Australia with a shared objective to ensure and support a “free,
open and prosperous” Indo-Pacific region.

The idea of Quad was first mooted by Japanese Prime Minister Shinzo Abe in 2007. However,
the idea couldn’t move ahead with Australia pulling out of it, apparently due to Chinese
pressure.

www.insightsactivelearn.com 44
Total Marks : 200
Test-2 (Subject)
( INSTA Prelims Test Series 2021 )

In December 2012, Shinzo Abe again floated the concept of Asia’s “Democratic Security
Diamond” involving Australia, India, Japan and the US to safeguard the maritime commons
from the Indian Ocean to the western Pacific.

https://www.thehindubusinessline.com/opinion/can-the-quad-rise-to-be-an-asian-nato/article32
103609.ece

57. Which of the following is/are the characteristics of Carnatic Music?


1. It adheres to time
2. It is indigenous in nature
3. Instruments are equally important as vocals

Select the correct answer using the code given below


A. 2 only
B. 1 and 3 only
C. 3 only
D. 2 and 3 only

Correct Answer : A

Answer Justification :

The Carnatic branch creates music that is played in the traditional octave. The music is kriti
based and focuses more on the saahitya or the lyric quality of the musical piece. The Kriti is a
highly evolved musical song set to a certain raga and fixed tala or rhythmic cycle.

Characteristics of Carnatic Music

Indigenous
No freedom to improvise
Only one particular prescribed style of singing
More emphasis on vocal music
Doesn’t adhere to any time.

58. Consider the following statements regarding rock paintings


1. The first discovery of rock paintings was made in India by an archaeologist, Archibold
Carlleyle
2. The caves of Bhimbetka were discovered by eminent archaeologist V.S. Wakankar.

Which of the statements given above is/are correct?


A. 1 only
B. 2 only
C. Both 1 and 2
D. Neither 1 nor 2

www.insightsactivelearn.com 45
Total Marks : 200
Test-2 (Subject)
( INSTA Prelims Test Series 2021 )

Correct Answer : C

Answer Justification :

It is interesting to know that the first discovery of rock paintings was made in India
in 1867–68 by an archaeologist, Archibold Carlleyle, twelve years before the discovery of
Altamira in Spain. Cockburn, Anderson, Mitra and Ghosh were the early archaeologists who
discovered a large number of sites in the Indian sub-continent.

The caves of Bhimbetka were discovered in 1957–58 by eminent archaeologist V.S.


Wakankar and later on many more were discovered. Wakankar spent several years in
surveying these inaccessible hills and jungles to study these paintings.

Hence, option (c) is correct.

The themes of paintings found here are of great variety, ranging from mundane events of daily
life in those times to sacred and royal images. These include hunting, dancing, music, horse
and elephant riders, animal fighting, honey collection, decoration of bodies, and other
household scenes. The rock art of Bhimbetka has been classified into various groups on the
bases of style, technique and superimposition. The drawings and paintings can be catagorised
into seven historical periods. Period I, Upper Palaeolithic; Period II, Mesolithic; and Period III,
Chalcolithic.

The artists of Bhimbetka used many colours, including various shades of white, yellow, orange,
red ochre, purple, brown, green and black. But white and red were their favourite colours.

59. Which of the following Sanskrit plays were written by the king Harshavardhana?
1. Urubhanga
2. Ratnavali
3. Amuktamalyada

Select the correct answer using the code given below:


A. 1 and 2 only
B. 1 and 3 only
C. 2 only
D. 2 and 3 only

Correct Answer : C

www.insightsactivelearn.com 46
Total Marks : 200
Test-2 (Subject)
( INSTA Prelims Test Series 2021 )

Answer Justification :

Urubhanga is a Sanskrit play written by Bhasa. It focuses on the story of the character
Duryodhana during and after his fight with Bhima. Hence, statement 1 is incorrect.

Ratnavali is a Sanskrit drama about a beautiful princess named Ratnavali, and a great
king named Udayana. It is attributed to the Indian emperor Harsha. Apart from
Ratnavali, Harsha has written two more plays – “Priyadarshika” (union of Udayana and
Priyadarshika, daughter of King Dridhavarman) and “Naganand” (story of how prince
Jimutavahana gives up his own body to stop a sacrifice of serpents to the divine Garuda).
Hence, statement 2 is correct.

Amuktamalyada is an epic poem in Telugu composed by Krishnadevaraya of the


Vijayanagar Dynasty. Hence, statement 3 is incorrect.

60. Consider the following statements regarding Abanindranath Tagore


1. He first created the ‘Indian Society of Oriental Art’.
2. Most of his works revolved around Hindu philosophy.
3. Abanindranath started integrating Chinese and Japanese calligraphic traditions into his style.

Which of the statements given above is/are correct?


A. 2 and 3 only
B. 3 only
C. 2 only
D. 1, 2 and 3

Correct Answer : D

Answer Justification :

All the statements given above are correct.

Abanindranath Tagore, the nephew of Rabindranath Tagore, was one of the most prominent
artists of Bengal school of art in India. He was the first major supporter of swadeshi values in
Indian art.

The contribution of Abanindranath Tagore towards Indian art and culture are:

Bengal School of Art:

He first created the ‘Indian Society of Oriental Art’ and later went on to establish
Bengal school of art.

He believed that Indian art and its art forms gave importance to spirituality as opposed to the
West which stressed on materialism, thus rejecting it.

www.insightsactivelearn.com 47
Total Marks : 200
Test-2 (Subject)
( INSTA Prelims Test Series 2021 )

His idea of modernizing Mughal and Rajput paintings eventually gave rise to modern Indian
painting, which took birth at his Bengal school of art.

Most of his works revolved around Hindu philosophy.

In his later works, Abanindranath started integrating Chinese and Japanese


calligraphic traditions into his style. The intention behind this move was to construct an
amalgamation of the modern pan-Asian artistic tradition and the common elements of Eastern
artistic and spiritual culture.

61. Consider the following statements regarding Tappa


1. Rhythm plays a very important role as the compositions are based on fast, subtle and knotty
constructions.
2. It gained legitimacy as a semi-classical vocal specialty once it was brought to the Mughal court
of emperor Muhammad Shah.
3. It originated from the folk songs of the camel riders of North-West India

Which of the statements given above is/are correct?


A. 1 and 3 only
B. 2 only
C. 3 only
D. 1, 2 and 3

Correct Answer : D

Answer Justification :

All the statements given above are correct

Tappa

In this style the rhythm plays a very important role as the compositions are based on
fast, subtle and knotty constructions. It originated from the folk songs of the camel
riders of North-West India but it gained legitimacy as a semi-classical vocal specialty
once it was brought to the Mughal court of emperor Muhammad Shah.

There is a great use of very quick turn of phrases. Tappa was the genre of choice of the
wealthy elite as well as the classes with more modest means.

The “baithaki” style, evolved under the direct patronage of the landed elites of the zamindari
classes of the late 19th and early 20th centuries in their baithak-khanas (literally, baithak -
assembly, khana - halls or salons) and jalsaghar (literally, halls for entertainment, mujra or
nautch halls)

www.insightsactivelearn.com 48
Total Marks : 200
Test-2 (Subject)
( INSTA Prelims Test Series 2021 )

62. Consider the following statements regarding Ajanta Cave Paintings


1. Ajanta caves were carved in 4th century AD out of volcanic rocks.
2. The walls of the caves have both murals and fresco paintings.
3. The unique feature of the paintings is that all female figure has a same hairstyle.

Which of the statements given above is/are correct?


A. 1 and 2 only
B. 1 and 3 only
C. 2 and 3 only
D. 1, 2 and 3

Correct Answer : A

Answer Justification :

Ajanta Cave Paintings

One of the oldest surviving murals of Indian subcontinent, Ajanta caves were carved in 4th
century AD out of volcanic rocks. It consists of a set of 29 caves, carved in a horse-shoe
shape.

The walls of the caves have both murals and fresco paintings (painted on wet plaster). They
use tempera style, i.e. use of pigments. The paintings portray human values and social fabric
along with styles, costumes and ornaments of that period. The emotions are expressed through
hand gestures. The unique feature of the paintings is that each female figure has a unique
hairstyle. Hence, statement 3 is incorrect.

Even animals and birds are shown with emotions. The common themes of these paintings
range from Jataka stories to life of Buddha to elaborate decorative patterns of flora and
fauna. Graceful poses of humans and animals adorn the walls of the caves. The medium of
painting was vegetable and mineral dyes. The outline of the figures is red ochre, with
contours of brown, black or deep red.

63. Consider the following statements regarding the various Buddhist texts:
1. The Visuddhimagga was written by Buddhaghosa.
2. Udanavarga contains utterances of Buddha and his disciples was written in Pali Language.
3. Abhidharmamoksha contains the famous story of “Blind Men and the Elephant”.

Which of the statements given above is/are correct?


A. 1 and 2 only
B. 1 only
C. 3 only
D. 1, 2 and 3

www.insightsactivelearn.com 49
Total Marks : 200
Test-2 (Subject)
( INSTA Prelims Test Series 2021 )

Correct Answer : B

96 pp tre

4
37 n
, 9 tsa en
Answer Justification :

04
86 o
55 ha k C
32 W oo
Visuddhimagga was written by Buddhagosha is a text of Theravada doctrine which

40 or B
contains discussions on various teachings of Buddha. Hence, statement 1 is correct.

26 all ar
88 C um
The Udanavarga is an early Buddhist collection of "utterances" attributed to the

K
Buddha and his disciples. Hence, statement 2 is incorrect.

Udana was a Buddhist text which contains the famous story of “Blind Men
and the Elephant”. Hence, statement 3 is incorrect.

Abhidharmamoksha was written by Vasubandhu in Sanskrit language containing the


discussion on Abhidharma.

64. Consider the following statements regarding the Shanghai Cooperation Organisation (SCO):
1. It is a permanent intergovernmental international organization.
2. The Heads of State Council (HSC) is the supreme decision-making body in the SCO.
3. Recently, Republic of Nepal became member of the organisation.

Which of the statements given above is/are correct?


A. 1 and 2 only
B. 1 only
C. 2 and 3 only
D. 1, 2 and 3

Correct Answer : A

Answer Justification :

The Shanghai Cooperation Organisation (SCO) is a permanent intergovernmental


international organisation, the creation of which was announced on 15 June 2001 in
Shanghai (China) by the Republic of Kazakhstan, the People's Republic of China, the Kyrgyz
Republic, the Russian Federation, the Republic of Tajikistan, and the Republic of Uzbekistan.
It was preceded by the Shanghai Five mechanism. Hence, statement 1 is correct.

The Shanghai Cooperation Organisation Charter was signed during the St.Petersburg SCO
Heads of State meeting in June 2002, and entered into force on 19 September 2003. This is the
fundamental statutory document which outlines the organisation's goals and principles, as well
as its structure and core activities.

www.insightsactivelearn.com 50
Total Marks : 200
Test-2 (Subject)
( INSTA Prelims Test Series 2021 )

96 pp tre
The historical meeting of the Heads of State Council of the Shanghai Cooperation Organisation

4
37 n
, 9 tsa en
was held on 8-9 June 2017 in Astana. On the meeting the status of a full member of the

04
86 o
55 ha k C
Organization was granted to the Republic of India and the Islamic Republic of Pakistan.

32 W oo
40 or B
26 all ar
88 C um
The Heads of State Council (HSC) is the supreme decision-making body in the SCO. It

K
meets once a year and adopts decisions and guidelines on all important matters of
the organisation. The SCO Heads of Government Council (HGC) meets once a year to discuss
the organisation's multilateral cooperation strategy and priority areas, to resolve current
important economic and other cooperation issues, and also to approve the organisation's
annual budget. The SCO's official languages are Russian and Chinese. Hence, statement 2 is
correct.

Presently, the SCO comprises eight member states, namely the Republic of India, the
Republic of Kazakhstan, the People's Republic of China, the Kyrgyz Republic, the
Islamic Republic of Pakistan, the Russian Federation, the Republic of Tajikistan, and
the Republic of Uzbekistan;

Hence, statement 3 is incorrect.

The SCO counts four observer states, namely the Islamic Republic of Afghanistan, the
Republic of Belarus, the Islamic Republic of Iran and the Republic of Mongolia;

The SCO has six dialogue partners, namely the Republic of Azerbaijan, the Republic
of Armenia, the Kingdom of Cambodia, the Federal Democratic Republic of Nepal, the
Republic of Turkey, and the Democratic Socialist Republic of Sri Lanka.

https://www.pib.gov.in/PressReleasePage.aspx?PRID=1640916

65. Consider the following statements regarding Ghazal


1. It is said to be originated in Ghazni, Afghanistan
2. The Ghazal spread in South Asia in the 12th century due to the influence of Sufi mystics

Which of the statements given above is/are correct?


A. 1 only
B. 2 only
C. Both 1 and 2
D. Neither 1 nor 2

Correct Answer : B

www.insightsactivelearn.com 51
Total Marks : 200
Test-2 (Subject)
( INSTA Prelims Test Series 2021 )

Answer Justification :

Ghazal

It is a poetic form that consists of rhyming couplets and a refrain, with each line sharing the
same meter. A Ghazal may be understood as a poetic expression of both the pain of loss or
separation and the beauty of love in spite of that pain. It is said to have originated in Iran
in the 10th century AD. A Ghazal never exceeds the 12 ashaar or couplets. Hence
Statement 1 is incorrect.

The Ghazal spread in South Asia in the 12th century due to the influence of Sufi mystics and
the courts of the new Islamic Sultanate, but it reached its zenith in the Mughal period. Hence
Statement 2 is correct.

66. Consider the following statements regarding Armamalai Cave Paintings


1. These natural caves were converted into Hindu temple in 8th century.
2. The paintings depict the tales of Astathik Palakas (deities protecting eight corners) and
Jainism.

Which of the statements given above is/are correct?


A. 1 only
B. 2 only
C. Both 1 and 2
D. Neither 1 nor 2

Correct Answer : B

Answer Justification :

Armamalai Cave Paintings

Situated in Vellore district of Tamil Nadu, these natural caves were converted into Jain
temple in 8th century. Hence, statement 1 is incorrect.

Unbaked mud structures are located within the cave, which acted like the place of rest for
Jain saints. The beautiful colourful paintings on the walls and roof depict the tales of Astathik
Palakas (deities protecting eight corners) and Jainism.

67. Pampa, Ponna and Ranna were considered as Triratnas (three gems) of:

A. Marathi Literature
B. Telugu Literature
C. Kannada Literature

www.insightsactivelearn.com 52
Total Marks : 200
Test-2 (Subject)
( INSTA Prelims Test Series 2021 )

D. Sanskrit Literature

Correct Answer : C

Answer Justification :

Pampa, Ponna and Ranna are called the "three gems" of Kannada literature.

Pampa was considered as one of the greatest Kannada writers. He became famous for
Adipurana (It is the life history of the first Jain thirtankara Rishabhadeva). His other classic
Vikramarjuna Vijaya or Pampa Bharata, is the modified version of the Hindu epic the
Mahabharata.

Ponna was patronised by King Krishna III and known for his description of the life of the
16th Jain tirthankara Shantinatha entitled Santipurana.

Ranna was court poet of Western Chalukya King Tailapa II. He was mainly known for
Ajitha purana and Saahasabhima Vijaya (also known as Gadaayuddha.

Hence, option (c) is correct.

68. Consider the following statements regarding the Hydrogen-Enriched Compressed Natural Gas (HCNG):
1. It may be used as a fuel of Internal Combustion Engine (ICE).
2. It reduces emissions of Carbon Monoxide (CO).

Which of the statements given above is/are correct?


A. 1 only
B. 2 only
C. Both 1 and 2
D. Neither 1 nor 2

Correct Answer : C

Answer Justification :

Both the statements given above are correct.

Hydrogen-Enriched Compressed Natural Gas (HCNG) is a mixture of compressed natural gas


(CNG) and some % Hydrogen by energy.

HCNG which may be used as a fuel of Internal Combustion Engine (ICE) is considered
a cleaner source of fuel, more powerful and offers more mileage then even CNG.

Advantages of HCNG:

www.insightsactivelearn.com 53
Total Marks : 200
Test-2 (Subject)
( INSTA Prelims Test Series 2021 )

HCNG reduces emissions of CO up to 70%.


Enables up to 5 % savings in fuel.
First step towards future Hydrogen economy.
Engines can be calibrated to release lower amounts of NO.
Engines need minimum modification to run on HCNG.
Ideal fuel for high load applications and heavy-duty vehicles.
Better performance due to higher Octane rating of H2.

Disadvantages of using HCNG:

Determining the most optimized H2/ NG (Natural Gas) ratio.


It requires new infrastructures for preparing HCNG.
Many steps need to be taken for commercializing it at a large scale.
Current cost of H2 is more than the cost of Natural Gas. So, HCNG’s cost is more than
CNG.

https://pib.gov.in/PressReleasePage.aspx?PRID=1640553

https://www.financialexpress.com/auto/industry/cars-could-soon-run-on-hydrogen-cng-as-per-g
ovt-plans-benefits-explained-hcng-dtc-bus-uber-ola-cabs-hydrogen-fuel-cell-hev-cost-
composition/2033677/

69. Which of the musical masters were employed and patronized by Akbar?
1. Gopal Das
2. Chhajju Khan
3. Tansen

Which of the statements given above is/are correct?


A. 1 and 3 only
B. 2 only
C. 2 and 3 only
D. 1, 2 and 3

Correct Answer : A

Answer Justification :

Akbar employed and patronised musical masters like Baba Gopal Das, Swami Haridas

www.insightsactivelearn.com 54
Total Marks : 200
Test-2 (Subject)
( INSTA Prelims Test Series 2021 )

and Tansen, who was considered to be one of the Navaratna or nine gems of the Mughal
court. It is also speculated that Baiju Bawra sang at the court of Akbar.

Bhendibazaar Gharana: Chhajju Khan, Nazir Khan and Khadim Hussain Khan founded it
in the 19th century. They gained popularity and fame as the singers were trained to control
their breath for a long period. Hence statement 2 is incorrect.

70. Consider the following statements regarding Lepakshi Paintings


1. These mural paintings were executed on Veerabhadra temple walls at Lepakshi.
2. The paintings show the dominance of primary colours, especially blue.

Which of the statements given above is/are correct?


A. 1 only
B. 2 only
C. Both 1 and 2
D. Neither 1 nor 2

Correct Answer : A

Answer Justification :

Lepakshi Paintings

Located in the Anantapur district of Andhra Pradesh, these mural paintings were executed
on Veerabhadra temple walls at Lepakshi in 16th Century. Made during the Vijaynagara
period, they follow a religious theme, based on Ramayana, Mahabharata and incarnations of
Vishnu. The paintings show a complete absence of primary colours, especially blue.
Hence, statement 2 is incorrect.

They depict a decline in painting in terms of quality. The forms, figures and details of their
costumes are outlined with black colour.

71. Consider the following statements regarding the development of mathematics during ancient India:
1. Extraction of square and cube roots of large numbers can be found in the text Aryabhatiya.
2. The Sanskrit name for zero-dot is Sunya-bindu which was clearly stated in Subandhu‘s
Vasavadatta

Which of the statements given above is/are correct?


A. 1 only
B. 2 only
C. Both 1 and 2
D. Neither 1 nor 2

www.insightsactivelearn.com 55
Total Marks : 200
Test-2 (Subject)
( INSTA Prelims Test Series 2021 )

Correct Answer : C

Answer Justification :

Both statements given above are correct.

Aryabhata, in his book Aryabhatiya presented his astronomical and mathematical theories. He
took the earth to rotate on its axis and he gave planet periods with respect to the sun.

In India, the method of extraction of square and cube roots of large numbers first
appeared in the Aryabhatiya (A.D. 499). This was followed by Brahmagupta (A.D. 598)
who, however, did not give any rule for square root extraction.

Vasavadatta is a classical Sanskrit romantic tale, written in an ornate style of language. Its
author is Subandhu

72. Consider the following statements regarding the recently launched Dhruvastra:
1. It is an anti-tank guided missile (ATGM) system.
2. It was developed by the Defence Research and Development Organisation (DRDO).

Which of the statements given above is/are correct?


A. 1 only
B. 2 only
C. Both 1 and 2
D. Neither 1 nor 2

Correct Answer : C

Answer Justification :

Both the statements given above are correct.

Recently, India has tested a locally developed anti-tank guided missile (ATGM)
system called Dhruvastra.

The ATGM underwent three flight trials at the Integrated Test Range in Odisha’s Chandipur.

Developed by the Defence Research and Development Organisation (DRDO), the


helicopter-launched Dhruvastra is designed to destroy enemy battle tanks, armoured

www.insightsactivelearn.com 56
Total Marks : 200
Test-2 (Subject)
( INSTA Prelims Test Series 2021 )

vehicles and bunkers.

Dhruvastra is designed as a customised and updated version of the ‘Nag Helina’ range
of DRDO’s anti-tank destroyers.

73. Consider the following statements regarding Sangeet Natak Academy

1. It was the first national academy set up for the Arts by the Government of India in 1952.
2. It has established several institutions like Manipur Dance Academy and Sattriya Centre.
3. It gives grants to aid research, documentation and publishing in the performing arts.

Which of the statements given above is/are correct?


A. 3 only
B. 1, 2 and 3
C. 1 and 2 only
D. 2 only

Correct Answer : B

Answer Justification :

All the statements given above are correct.

The Sangeet Natak Academy was the first national academy set up for the Arts by the
Government of India in 1952. The major focus of the academy was to create a set-up for
music, drama and dances in India.

The academy functions as the apex body of the performing arts in the country to preserve and
promote the vast cultural heritage of India expressed in music, dance and drama. It also works
with governments and art academies in states and territories of the country.

SNA established several institutions over the years:

Manipur Dance Academy, Imphal


Sattriya Centre
Kathak Kendra (National Institute of Kathak Dance), New Delhi in 1964
Ravindra Rangshala

In addition, the Akademi

Subsidizes the work of institutions engaged in teaching, performing or promoting music,


dance, or theatre
Gives grants to aid research, documentation and publishing in the performing
arts
Organises and subsidises seminars and conferences of subject specialists

www.insightsactivelearn.com 57
Total Marks : 200
Test-2 (Subject)
( INSTA Prelims Test Series 2021 )

74. Consider the following statements regarding paintings under Shah Jahan
1. He established a formal artistic studio called Tasvir Khana.
2. Shah Jahan liked to create artificial elements in the paintings.
3. He ordered to increase the use of gold and silver in the paintings.

Which of the statements given above is/are correct?


A. 1 and 2 only
B. 1 and 3 only
C. 2 and 3 only
D. 1, 2 and 3

Correct Answer : C

Answer Justification :

Akbar

Akbar was responsible for establishment of an entire department devoted to the paintings
and scribing of his documents. He established a formal artistic studio called Tasvir Khana
where the artists were hired on salary and they developed their own styles. Hence,
statement 1 is incorrect.

Shah Jahan

The tenor of the Mughal paintings changed rapidly in the period of Shah Jahan. Unlike his
father and grandfather who liked naturalistic depictions, Shah Jahan liked to create artificial
elements in the paintings. It is said that he tried to reduce the liveliness of the paintings
and bring in unnatural stillness as he was inspired by the European influence.

He brought a change in the technique of drawing and painting from the earlier period. He
eschewed the use of charcoal to draw and encouraged the artists to draw and sketch using
a pencil. He also ordered to increase the use of gold and silver in the paintings.

75. Which of the following communities does/do celebrate the Sarhul festival?
1. Todas
2. Badagas
3. Soligas

Select the correct answer using the code given below:

www.insightsactivelearn.com 58
Total Marks : 200
Test-2 (Subject)
( INSTA Prelims Test Series 2021 )

A. 3 only
B. 1 and 2 only
C. 1, 2 and 3
D. None

Correct Answer : D

Answer Justification :

None of the above given tribal communities are associated with Sarhul Festival.

Sarhul is spring festival celebrated during spring season when the Saal trees get new
flowers on their branches. It is a worship of the village deity who is considered to be the
protector of the tribes. It marks the beginning of New Year and celebrated by the Oraon,
the Munda and the Ho tribes, of the Jharkhand region.

76. Kakrapar Atomic Power Plant is located in the state of:

A. Maharashtra
B. Rajasthan
C. Gujarat
D. Karnataka

Correct Answer : C

Answer Justification :

The Kakrapar Atomic Power Station (KAPS) is located near Surat in the state of
Gujarat. KAPS currently operates two Pressurized Heavy Water Reactors (PHWR). The
facilities fall under the direction of the Nuclear Power Corporation of India, Limited (NPCIL)

https://indianexpress.com/article/explained/kakrapar-atomic-power-project-third-unit-
achieves-first-criticality-india-nuclear-mission-6518946/

https://pib.gov.in/PressReleasePage.aspx?PRID=1640338

77. A community of people called Manganiyars is well-known for their

A. Martial arts in North-East India


B. Musical tradition in North-West India

www.insightsactivelearn.com 59
Total Marks : 200
Test-2 (Subject)
( INSTA Prelims Test Series 2021 )

C. Classical vocal music in South India


D. Pietra dura tradition in Central India

Correct Answer : B

Answer Justification :

Manganiyars is well-known for their Musical tradition in North-West India. Famous for
their classical folk music, the Manganiyars find a special mention and inordinate respect in the
folk music circuit of India. Manganiyar is a hereditary community living in the obscure villages
between India and Pakistan.

In the Sindh province of Pakistan and in India’s Western Rajasthan, the Manganiars have been
musical performers for generations and genealogists for higher caste patrons. These skilled
folk musicians of the Thar pass on their songs from generation to generation in a form of oral
history of the desert.

https://www.thehindu.com/entertainment/music/why-manganiyars-love-the-kamaicha/article22
761362.ece

78. Consider the following statements regarding Basholi School


1. The paintings created in the Pahari School in the 17th century was called the Basholi School.
2. The characteristic feature was expressive faces with a receding hairline and big eyes that are
shaped like lotus petals.
3. The first patron of this school was Raja Kirpal Singh.

Which of the statements given above is/are correct?


A. 1 and 2 only
B. 1 and 3 only
C. 2 and 3 only
D. 1, 2 and 3

Correct Answer : D

Answer Justification :

Basholi School

The paintings created in the Pahari School in the 17th century were called the Basholi
School. This was the early phase and expressive faces with a receding hairline and big eyes
that are shaped like lotus petals characterise it. These paintings use a lot of primary
colours, i.e. red, yellow and green. They used the Mughal technique of painting on clothing
but developed their own styles and techniques. The first patron of this school was Raja Kirpal
Singh who ordered the illustration of Bhanudatta’s Rasamajari, Gita Govinda and the

www.insightsactivelearn.com 60
Total Marks : 200
Test-2 (Subject)
( INSTA Prelims Test Series 2021 )

Ramayana drawings. The most famous painter of this school was Devi Das who was famous
for his depiction of Radha Krishna and the portrait of kings in their livery and in white
garments. The contrast of colours is associated with this school and they are borrowed from
the Malwa paintings.

79. Consider the following pairs of the Handicrafts and their respective states:
Handicraft: State
1. Telia Rumal Kerala
2. Kasuti embroidery Gujarat
3. Bhavani Jamakkalam Maharashtra

Which of the pairs given above is/are not correctly matched?


A. 1 and 2 only
B. 2 and 3 only
C. 1, 2 and 3
D. 1 only

Correct Answer : C

Answer Justification :

None of the pairs given above are correctly matched.

Telia Rumal is a method for the oil treatment of yarn. It originated from Chirala in
Andhra Pradesh.

Kasuti is a traditional form of folk embroidery practised in the state of Karnataka,


India.

Bhavani Jamakkalam refers to blankets and carpets manufactured in Bhavani in


Erode district, Tamil Nadu. It has been recognized as a Geographical indication (accorded
GI Tag) by the Government of India in 2005-06.

80. Consider the following statements regarding the AT1 (additional tier 1) bonds:
1. They are a special category of debt issued by banks.
2. They are designed to absorb losses in case the bank’s equity capital dips below a certain
threshold.
3. They have a fixed maturity date of 36 months.

Which of the statements given above is/are correct?

www.insightsactivelearn.com 61
Total Marks : 200
Test-2 (Subject)
( INSTA Prelims Test Series 2021 )

A. 1 and 2 only
B. 2 and 3 only
C. 1 and 3 only
D. 1, 2 and 3

Correct Answer : A

Answer Justification :

AT1 are a special category of debt issued by banks. They are designed to absorb
losses in case the bank’s equity capital dips below a certain threshold. Hence,
statement 1 and statement 2 are correct.

Banks can stop paying interest on them and also write off their value. They do not have a
fixed maturity date but a bank can repay them after specific tenures by exercising a
call option on them. Hence, statement 3 is incorrect.

Under this call option, the bank has the right but not the obligation to pay back the principal
amount on the specified dates. If the bank chooses to not repay the principal, it can carry on
paying the interest on them in perpetuity.

https://m.timesofindia.com/business/india-business/yes-bank-writes-down-rs-8415-crore-at-1-bo
nds/amp_articleshow/74643881.cms

81. Consider the following statements regarding Kambala


1. It is traditional slush track buffalo race held annually in coastal districts of Kerala
2. Slushy/marshy paddy field track is used for Kambala
3. It was recently banned by Central government

Which of the statements given above is/are correct?


A. 2 only
B. 1 and 3 only
C. 2 and 3 only
D. 3 only

Correct Answer : A

Answer Justification :

It is traditional slush track buffalo race held annually in coastal districts of


Karnataka to entertain rural people of the area. Hence Statement 1 is incorrect.

www.insightsactivelearn.com 62
Total Marks : 200
Test-2 (Subject)
( INSTA Prelims Test Series 2021 )

Traditionally, it is sponsored by local Tuluva landlords and households in the coastal districts
of Dakshina Kannada and Udupi of Karnataka and Kasaragod of Kerala.

Slushy/marshy paddy field track is used for Kambala. Kambla in its traditional form is
non-competitive, which is considered a thanksgiving to the Gods for protecting the animals
from diseases. Hence Statement 2 is correct.

It has not been banned by Central Government. Hence Statement 3 is incorrect.

82. Consider the following statements regarding Bazaar Paintings


1. The Bazaar school of Painting had no Indian influence.
2. They made the painters to copy the Greek and Roman statues.
3. This school was prevalent in the Bengal and Bihar region.

Which of the statements given above is/are correct?


A. 1 and 2 only
B. 1 and 3 only
C. 2 and 3 only
D. 1, 2 and 3

Correct Answer : D

Answer Justification :

Bazaar Paintings

This school was also influenced by the European encounter in school mixed European
techniques and themes with Indian ones. The Bazaar school did not take any Indian
influence but took the Roman and Greek influence. They made the painters to copy the
Greek and Roman statues.

This school was prevalent in the Bengal and Bihar region. Apart from the Greco-roman
heritage, they made paintings on everyday bazaar that showed Indian bazaars with European
background. They also painted religious themes but the figures of Indian Gods and
Goddesses with more than two axes and elephant faces like that of Lord Ganesha were
prohibited as they deviated from the European notion of natural human figurine.

83. Consider the following pairs of fairs in India and their respective states:
Fairs: State:
1. Surajkund Crafts Fair Haryana
2. Pushkar Mela Rajasthan
3. Shamlaji Fair Gujarat

www.insightsactivelearn.com 63
Total Marks : 200
Test-2 (Subject)
( INSTA Prelims Test Series 2021 )

Which of the pairs given above is/are correctly matched?


A. 1 and 2 only
B. 2 and 3 only
C. 1 and 3 only
D. 1, 2 and 3

Correct Answer : D

Answer Justification :

All the pairs given above are correctly matched.

The Surajkund Mela is unique as it showcases the richness and diversity of the handicrafts,
handlooms and cultural fabric of India, and is the largest crafts fair in the world. It is
celebrated in Surajkund, Faridabad district of Haryana. The Mela is organized by the
Surajkund Mela Authority & Haryana Tourism in collaboration with Union Ministries
of Tourism, Textiles, Culture and External Affairs.

Pushkar Mela is an annual fair in Rajasthan starting on the day of ‘Kartik Poornima’ and
lasts for about a week. It is one of the largest camel and cattle fairs in the world.

The Shamlaji Mela is held at Shamlaji situated in the Sabarkantha district, Gujarat.
Among the Adivasis, the Bhil community has incredible faith in the powers of
Shamlaji who they lovingly refer to as Kaliyo Dev (Dark Divinity). The fair is an
important trade destination for people to buy silver ornaments, metalware, cloth and
garments, along with many other annual household items. The temple of Shamlaji stands
on the banks of the river Meshwo, in veneration of Lord Vishnu.

84. Consider the following statements regarding the Consumer Protection Act, 2019:
1. The Act includes establishment of the Central Consumer Protection Authority (CCPA) to
promote, protect and enforce the rights of consumers.
2. The Act introduces the concept of product liability.
3. The Act provides protection to buyers from both traditional sellers and the new e-commerce
retailers/platforms.

Which of the following statements given above is/are correct?


A. 1 and 2 only
B. 2 and 3 only
C. 1 and 3 only
D. 1, 2 and 3

Correct Answer : D

www.insightsactivelearn.com 64
Total Marks : 200
Test-2 (Subject)
( INSTA Prelims Test Series 2021 )

Answer Justification :

All the statements given above are correct.

The Consumer Protection Act, 2019 comes in to force from 20th July 2020.

The Act includes establishment of the Central Consumer Protection Authority (CCPA)
to promote, protect and enforce the rights of consumers. The CCPA will be empowered
to conduct investigations into violations of consumer rights and institute complaints /
prosecution, order recall of unsafe goods and services, order discontinuance of unfair trade
practices and misleading advertisements, impose penalties on
manufacturers/endorsers/publishers of misleading advertisements.

The new Act also introduces the concept of product liability and brings within its
scope, the product manufacturer, product service provider and product seller, for any
claim for compensation. The Act provides for punishment by a competent court for
manufacture or sale of adulterant/spurious goods. The court may, in case of first conviction,
suspend any licence issued to the person for a period of up to two years, and in case of second
or subsequent conviction, cancel the licence.

The new act has been introduced after many amendments to provide protection to
buyers not only from traditional sellers but also from the new e-commerce
retailers/platforms.

https://www.livemint.com/news/india/consumer-protection-act-2019-comes-into-force-from-tod
ay-covers-e-commerce-too-11595245073150.html

https://pib.gov.in/PressReleasePage.aspx?PRID=1639925

85. Consider the following statements regarding Hampi


1. It is a UNESCO world heritage site.
2. Hampi was the capital city for all the four dynasties that ruled Vijayanagara Empire since its
establishment in the year 1336 AD

Which of the statements given above is/are correct?


A. 1 only
B. 2 only
C. Both 1 and 2
D. Neither 1 nor 2

Correct Answer : C

Answer Justification :

www.insightsactivelearn.com 65
Total Marks : 200
Test-2 (Subject)
( INSTA Prelims Test Series 2021 )

Both the statements are correct

Hampi is a UNESCO world heritage site. It was a part of the Mauryan Empire back in
the third century BC.

Hampi was the capital city during the four different dynasties altogether in the
Vijayanagar city that came into existence in the year 1336 AD.

The Vijayanagara Empire reached unfathomable heights under the guidance of King
Krishnadeva Raya of the Tuluva Dynasty.

86. Consider the following statements regarding Mithila paintings


1. The figures in the painting are symbolic, for example, fish depicts good luck and fertility.
2. The paintings are three-dimensional.
3. The paintings are made depicting auspicious occasions like birth, marriage and festivals.

Which of the statements given above is/are correct?


A. 1 and 2 only
B. 1 and 3 only
C. 2 and 3 only
D. 1, 2 and 3

Correct Answer : B

Answer Justification :

Madhubani Paintings

Traditionally done by the women of villages around Madhubani town, it is also called Mithila
paintings. The art extends to the adjoining parts of Terai region in Nepal. The paintings have a
common theme and are usually drawn from religious motifs of the Hindus, including Krishna,
Rama, Durga, Lakshmi and Shiva. The figures in the painting are symbolic, for example, fish
depicts good luck and fertility.

The paintings are also made depicting auspicious occasions like birth, marriage and festivals.
Flowers, trees, animals, etc. are used to fill any gaps in the painting. Traditionally, these
were painted on walls using rice paste and vegetable colours on a base of cow dung and mud.
With time, the base changed to handmade paper, clothes and canvas, still the natural colours
were used. Since there is no shading, the paintings are two-dimensional. Hence,
statement 2 is incorrect.

Some of the common features of these paintings include double line border, bold use of
colours, ornate floral patterns and exaggerated facial features.

Since the art has remained confined to a specific geographical area, it has been given GI

www.insightsactivelearn.com 66
Total Marks : 200
Test-2 (Subject)
( INSTA Prelims Test Series 2021 )

(geographical indication) status.

87. Consider the following statements regarding the various Literary Awards:
1. The Saraswati Samman is an annual award for outstanding prose or poetry literary works in
any of the 22 Indian languages listed in Schedule VIII of the Constitution of India.
2. Jnanpith award is not given posthumously.
3. The Vyas Samman is given by the Sahitya Akademi.

Which of the statements given above is/are correct?


A. 1 and 2 only
B. 2 only
C. 1 and 3 only
D. 1, 2 and 3

Correct Answer : A

Answer Justification :

The Saraswati Samman is an annual award for outstanding prose or poetry literary
works in any of the 22 Indian languages listed in Schedule VIII of the Constitution of
India. Hence, statement 1 is correct.

Jnanpith award is given for outstanding literary achievement. It is given to those Indian
citizens who compose literature in one of the 22 languages listed in the Schedule VIII of the
Indian Constitution and English. This award is not given posthumously. Hence,
statement 2 is correct.

The Vyas Samman is a literary award in India, first awarded in 1991. It is awarded
annually by the K. K. Birla Foundation. To be eligible for the award, the literary work must
be in Hindi language and has been published in the past 10 years. Hence, statement 3 is
incorrect.

88. Consider the following statements regarding the Psoriasis disease:


1. It is a genetic skin disease associated with the immune system.
2. It is highly contagious disease.

Which of the statements given above is/are correct?


A. 1 only
B. 2 only
C. Both 1 and 2
D. Neither 1 nor 2

www.insightsactivelearn.com 67
Total Marks : 200
Test-2 (Subject)
( INSTA Prelims Test Series 2021 )

Correct Answer : A

Answer Justification :

Psoriasis is a genetic skin disease associated with the immune system. Your immune
system causes your skin cells to reproduce too quickly. A normal skin cell matures and
falls off the body's surface in 28 to 30 days. However, skin affected by psoriasis takes only
three to four days to mature and move to the surface. Instead of falling off (shedding), the cells
pile up and form lesions. The skin also becomes very red due to increased blood flow.

Hence, statement 1 is correct.

Psoriasis is not contagious. It is not something you can "catch" or "pass on." The lesions
may not look good, but they are not infections or open wounds. People with psoriasis pose
no threat to the health or safety of others. Hence, statement 2 is incorrect.

https://www.thehindu.com/sci-tech/health/the-hindu-explains-why-is-a-psoriasis-drug-being-use
d-to-treat-covid-19-patients/article32126359.ece

89. Which one of the following is not a part of early Jain literature?

A. Therigatha
B. Acarangasutra
C. Sutrakritanga
D. Brihatkalpasutra

Correct Answer : A

Answer Justification :

Theragatha/Therigatha, Buddhist lyrics, included in the Suttanipata (one of the


earliest books of the Pali canon, appearing in the late Khuddaka Nikaya [“Short
Collection”] of the Sutta Pitaka).

In the works 264 monks speak of their inner experiences and of nature, and some 100 nuns tell
of their daily lives. The songs of the monks are said to have been composed when their authors
experienced the bliss of enlightenment. Within the collection about 30 different meters can be
distinguished, attesting to the prosodic variety of Buddhist lyrics.

90. Consider the following statements regarding Paitkar Painting


1. It is practiced by the tribal people of Madya Pradesh
2. The common theme of Paitkar paintings is ‘What happens to human life post death’.

www.insightsactivelearn.com 68
Total Marks : 200
Test-2 (Subject)
( INSTA Prelims Test Series 2021 )

Which of the statements given above is/are correct?


A. 1 only
B. 2 only
C. Both 1 and 2
D. Neither 1 nor 2

Correct Answer : B

Answer Justification :

Paitkar Painting

Practiced by the tribal people of Jharkhand, Paitkar paintings or scroll paintings are
considered one of the ancient schools of painting in the country. Hence, statement 1 is
incorrect.

This old form of painting has cultural association with Ma Mansa, one of the most popular
goddesses in tribal household.

These paintings are linked to the social and religious customs including giving alms and
holding yajnas. The common theme of Paitkar paintings is ‘What happens to human life
post death’. Although an ancient art, it is on the verge of extinction given the rate of its
decline.

91. The Archaeological Survey of India (ASI), works under the aegis of

A. Ministry of Tourism
B. Ministry of Culture
C. Prime Minister’s Office (PMO)
D. None

Correct Answer : B

Answer Justification :

The Archaeological Survey of India is an Indian government agency attached to the


Ministry of Culture that is responsible for archaeological research and the
conservation and preservation of cultural monuments in the country. It was founded
in 1861 by Alexander Cunningham who also became its first Director-General.

92. Consider the following statements regarding the United Nations Economic and Social Council (ECOSOC):
1. It is one of the principal organs of the United Nations.

www.insightsactivelearn.com 69
Total Marks : 200
Test-2 (Subject)
( INSTA Prelims Test Series 2021 )

2. ECOSOC has 54 member Governments which are elected for three-year terms by the General
Assembly
3. It is responsible for the follow-up to major UN conferences and summits.

Which of the statements given above is/are correct?


A. 1 and 2 only
B. 2 and 3 only
C. 1 and 3 only
D. 1, 2 and 3

Correct Answer : D

Answer Justification :

All the statements given above are correct.

The Economic and Social Council is at the heart of the United Nations system to advance the
three dimensions of sustainable development – economic, social and environmental. It is the
central platform for fostering debate and innovative thinking, forging consensus on ways
forward, and coordinating efforts to achieve internationally agreed goals. It is also
responsible for the follow-up to major UN conferences and summits.

The UN Charter established ECOSOC in 1945 as one of the six main organs of the
United Nations.

ECOSOC has 54 member Governments which are elected for three-year terms by the
General Assembly. Seats on the Council are allotted based on geographical representation
with fourteen allocated to African States, eleven to Asian States, six to Eastern European
States, ten to Latin American and Caribbean States, and thirteen to Western European and
other States.

https://www.livemint.com/news/india/in-india-we-have-made-fight-against-covid-19-a-people-s-
movement-pm-at-un-ecosoc-11594998832775.html

93. Consider the following pairs


Art Form Largely practiced in
1. Garodas Eastern India
2. Oja-Pali Assam
3. Kariyila Himachal Pradesh

Which of the pairs given above is/are matched correctly?


A. 2 and 3 only

www.insightsactivelearn.com 70
Total Marks : 200
Test-2 (Subject)
( INSTA Prelims Test Series 2021 )

B. 1 only
C. 1 and 3 only
D. 1, 2 and 3

Correct Answer : A

Answer Justification :

Kariyila is one of the forms of open-air theatre that is popular in the foothills of
Himachal Pradesh. Generally staged during village fairs and festivals, the performance
occurs through the night and comprises of a series of small plays and skits.

Garodas is a popular art form of the ‘Garoda’ community of Gujarat. It uses painted
pictures to narrate stories of romance and valour.

Oja-Pali is a unique narrative theatre form of Assam, primarily associated with the
festival of Manasa or the serpent goddess. The narration is a prolonged affair, with three
distinct parts – Baniya Khanda, Bhatiyali Khanda and Deva Khanda. Oja is the main narrator
and Palis are the members of the chorus.

94. Consider the following statements regarding Seals during Indus civilization
1. The purpose of producing seals was mainly commercial.
2. The standard Harappan seal was a square plaque 2×2 square inches.
3. The seals were mostly made of steatite.

Which of the statements given above is/are correct?


A. 1 and 2 only
B. 1 and 3 only
C. 2 and 3 only
D. 1, 2 and 3

Correct Answer : D

Answer Justification :

Seals

Archaeologists have discovered thousands of seals, mostly made of steatite, and


occasionally of agate, chert, copper, faience and terracotta, with beautiful figures of
animals, such as unicorn bull, rhinoceros, tiger, elephant, bison, goat, buffalo, etc. The
realistic rendering of these animals in various moods is remarkable. The purpose of
producing seals was mainly commercial. It appears that the seals were also used as
amulets, carried on the persons of their owners, perhaps as modern-day identity cards. The
standard Harappan seal was a square plaque 2×2 square inches, made from steatite.

www.insightsactivelearn.com 71
Total Marks : 200
Test-2 (Subject)
( INSTA Prelims Test Series 2021 )

Every seal is engraved in a pictographic script which is yet to be deciphered. Some seals have
also been found in ivory. They all bear a great variety of motifs, most often of animals
including those of the bull, with or without the hump, the elephant, tiger, goat and also
monsters. Sometimes trees or human figures were also depicted. The most remarkable seal is
the one depicted with a figure in the centre and animals around. This seal is generally
identified as the Pashupati Seal by some scholars whereas some identify it as the female deity.
This seal depicts a human figure seated cross-legged. An elephant and a tiger are depicted to
the right side of the seated figure, while on the left a rhinoceros and a buffalo are seen.

95. Consider the following statements regarding the Indian Council of Historical Research (ICHR):
1. It is an autonomous organization.
2. It was established under Societies Registration Act, 1860.

Which of the statements given above is/are correct?


A. 1 only
B. 2 only
C. Both 1 and 2
D. Neither 1 nor 2

Correct Answer : C

Answer Justification :

Both the statements given above are correct.

Indian Council of Historical Research is an autonomous organization which was


established under Societies Registration Act (Act XXI of 1860) in 1972. The objectives
of the Indian Council of Historical Research (hereafter referred to as the 'Council') as laid
down in the Memorandum of Association are as follows:

- "to bring historians together and provide a forum for exchange of views between them;

- to give a national direction to an objective and scientific writing of history and to have
rational presentation and interpretation of history;

- to promote, accelerate and coordinate research in history with special emphasis on areas
which have not received adequate attention so far;

- to promote and coordinated a balanced distribution of research effort over different areas;

- to elicit support and recognition for historical research from all concerned and ensure the
necessary dissemination and use of results."

96. Consider the following statements regarding the development in the field of Astronomy during Medieval period:

www.insightsactivelearn.com 72
Total Marks : 200
Test-2 (Subject)
( INSTA Prelims Test Series 2021 )

1. Mehendra Suri, developed an astronomical instrument ‘Yantraja’.


2. Nilakantha Somayaji produced commentary on Aryabhatiyaa.
3. Maharaja Sawai Jai Singh-II of Jaipur set up the astronomical observatories.

Which of the statements given above is/are correct?


A. 1 and 2 only
B. 2 and 3 only
C. 1 and 3 only
D. 1, 2 and 3

Correct Answer : D

Answer Justification :

All the statements given above are correct.

Mahendra Suri is the 14th century Jain astronomer who wrote the Yantraraja, the
first Indian treatise on the astrolabe. He was a court astronomer of Emperor Firoz Shah,
developed an astronomical instrument ‘Yantraja’

Nilakantha Somayaji was a major mathematician and astronomer of the Kerala School of
astronomy and mathematics. One of his most influential works was the comprehensive
astronomical treatise Tantrasamgraha completed in 1501. He had also composed an
elaborate commentary on Aryabhatiya called the Aryabhatiya Bhasya. In this Bhasya,
Nilakantha had discussed infinite series expansions of trigonometric functions and
problems of algebra and spherical geometry.

Maharaja Sawai Jai Singh-II of Jaipur was a patron of Astronomy. He set up the five
astronomical observatories in Delhi, Ujjain, Varansasi, Mathura and Jaipur.

97. Consider the following statements regarding Sufism


1. Sufis represent the inner side of the Islam creed.
2. It stresses on self-realization, tolerance, righteousness and universal love for all.
3. The Sufis tried to purify their inner self by rigid introspection and mental struggle.

Which of the statements given above is/are correct?


A. 2 and 3 only
B. 1 only
C. 1, 2 and 3
D. None

Correct Answer : C

www.insightsactivelearn.com 73
Total Marks : 200
Test-2 (Subject)
( INSTA Prelims Test Series 2021 )

Answer Justification :

All the statements given above are correct.

Sufism is the mystical arm of Islam. Sufis represent the inner side of the Islam creed
(tasawwuf).

It stresses on self realisation, tolerance, righteousness and universal love for all. The
word finds its roots in Arabic word for wool (suf), indicative of the garments of rough wool
worn by the ascetics and even the prophets. The word Sufism is sometimes also traced to root
word safā which in Arabic means purity.

Characteristics

The Sufis tried to purify their inner self by rigid introspection and mental struggle so
as to remove even the smallest sign of selfishness and attain ikhlās, absolute purity of
intention and act.

The mystics realized that beyond the knowledge of outward sciences, intuitive knowledge was
required in order to receive that illumination to which reason has no access. Dhawq, direct
“tasting” of experience, was essential for them

98. Consider the following statements


1. Sheshashayana is the form of Vishnu where he is shown reclining on the sheshanaga called
Ananta.
2. Nara-Narayan is the story of achieving moksha.
3. Gajendramoksha shows the discussion between the human soul and the eternal divine.

Which of the statements given above is/are correct?


A. 1 only
B. 1 and 3 only
C. 2 and 3 only
D. 1, 2 and 3

Correct Answer : A

Answer Justification :

Sheshashayana is the form of Vishnu where he is shown reclining on the sheshanaga


called Ananta.

Nara-Narayan shows the discussion between the human soul and the eternal divine.
Hence, statement 2 is incorrect.

Gajendramoksha is the story of achieving moksha, symbolically communicated by


Vishnu’s suppression of an asura who had taken the form of an elephant. Hence,
www.insightsactivelearn.com 74
Total Marks : 200
Test-2 (Subject)
( INSTA Prelims Test Series 2021 )

statement 3 is incorrect.

99. Consider the following statements:


1. Bhaskaracharya introduced negative numbers.
2. Mahaviracharya introduced Chakrawat Method or the Cyclic Method to solve algebraic
equations.
3. Brahmgupta wrote Ganit Sara Sangraha.

Which of the statements given above is/are correct?


A. 1 only
B. 2 only
C. 3 only
D. None

Correct Answer : D

Answer Justification :

None of the statements given above are correct.

Bhaskaracharya is famous for his book Siddanta Shiromani. It is divided into four sections:
Lilavati (Arithmetic), Beejaganit (Algebra), Goladhyaya (Sphere) and Grahaganit (mathematics
of planets). Bhaskara introduced Chakrawat Method or the Cyclic Method to solve
algebraic equations. This method was rediscovered six centuries later by European
mathematicians, who called it inverse cycle. In the nineteenth century, an English man,
James Taylor, translated Lilavati and made this great work known to the world.

In 7th century, Brahmgupta took mathematics to heights far beyond others. In his methods of
multiplication, he used place value in almost the same way as it is used today. He introduced
negative numbers and operations on zero into mathematics. He wrote Brahm Sputa
Siddantika

Jain Guru Mahaviracharya wrote Ganit Sara Sangraha in 850A.D., which is the first
textbook on arithmetic in present day form. The current method of solving Least common
Multiple (LCM) of given numbers was also described by him.

100. Consider the following statements regarding the State Disaster Response Fund (SDRF):
1. It was constituted under the Disaster Management Act, 2005.
2. Pest attack and Fire disasters are also covered under SDRF.
3. It shall be used only for meeting the expenditure for providing immediate relief to the victims.

Which of the statements given above is/are correct?


A. 1 and 2 only

www.insightsactivelearn.com 75
Total Marks : 200
Test-2 (Subject)
( INSTA Prelims Test Series 2021 )

B. 2 and 3 only
C. 1 and 3 only
D. 1, 2 and 3

Correct Answer : D

Answer Justification :

All the statements given above are correct.

The State Disaster Response Fund (SDRF), constituted under Section 48 (1) (a) of the
Disaster Management Act, 2005, is the primary fund available with State
Governments for responses to notified disasters. The Central Government contributes
75% of SDRF allocation for general category States/UTs and 90% for special category
States/UTs (NE States, Sikkim, Uttarakhand, Himachal Pradesh and erstwhile Jammu and
Kashmir). The annual Central contribution is released in two equal installments as per the
recommendation of the Finance Commission. SDRF shall be used only for meeting the
expenditure for providing immediate relief to the victims.

Disaster (s) covered under SDRF: Cyclone, drought, earthquake, fire, flood, tsunami,
hailstorm, landslide, avalanche, cloudburst, pest attack, frost and cold waves.

Local Disaster: A State Government may use up to 10 percent of the funds available under
the SDRF for providing immediate relief to the victims of natural disasters that they consider
to be ‘disasters’ within the local context in the State and which are not included in the notified
list of disasters of the Ministry of Home Affairs subject to the condition that the State
Government has listed the State specific natural disasters and notified clear and transparent
norms and guidelines for such disasters with the approval of the State Authority, i.e., the State
Executive Authority (SEC).

https://www.thehindu.com/news/national/govt-opens-up-ndrf-for-individual-donations/article32
125015.ece

www.insightsactivelearn.com 76

You might also like